Praxis 5422 Practice Exam 1 Flashcards

1
Q

Which of the following school counselor activities does NOT take place within the Manage component of the ASCA National Model?

a. Completing a use-of-time assessment

b. Analyzing a results report

c. Creating a closing-the-gap action plan

d. Writing a mission and vision statement

A

b. Analyzing a results report

Analyzing a results report is part of the Assess component of the ASCA National Model, not the Manage component. Results reports are used to evaluate the effectiveness of classroom instruction, small-group services to students, and closing-the-gap action plans. Completing a use-of-time assessment, creating a closing-the-gap action plan, and writing a mission and vision statement are all part of the effective management of a comprehensive school counseling program. These
activities help counselors clarify the needs of their school and students and define priorities for
their program

How well did you know this?
1
Not at all
2
3
4
5
Perfectly
2
Q

Solution-Focused Brief Counseling consists of what six steps?

a. Define problem, determine goals, develop intervention, assign strategic tasks, emphasizepositive behavior, terminate counseling

b. Define problem, determine goals, develop intervention, revisit goals, develop a second intervention, terminate counseling

c. Conduct psychoanalysis, determine underlying problem, develop group intervention plan, reinforce behavior, assign strategic task, terminate counseling

d. Identify prevention steps, define problem, determine goals, develop intervention, assign strategic task, terminate counseling

A

a. Define problem, determine goals, develop intervention, assign strategic tasks, emphasizepositive behavior, terminate counseling

Beginning a Solution-Focused Brief Counseling session involves having the student define the
problem and determine his desired goals of the counseling. With the assistance of the counselor, the
intervention or behavior change is both determined and initiated by the student. Strategic tasks are
assigned by the counselor and involve a direct behavior change. For example, if a student is having
difficulty with academics and has a goal to raise his grades, then a strategic task would be for him to
study for one hour every night. In subsequent counseling sessions, the counselor would offer support and positive feedback in order to encourage the student to reach his academic goals. Once the goals for that particular problem have been reached, the sessions are no longer required and counseling is terminated

How well did you know this?
1
Not at all
2
3
4
5
Perfectly
3
Q

What forms can professional development take?

a. Continued education

b. Evaluating and reporting on program outcomes

c. Presenting research at professional conferences

d. All of the above

A

d. All of the above

Continued education is required for professional licenses. Depending on the state board of education, a counselor will have a specific number of hours in continued education to complete. This education is intended to provide the counselor with the latest counseling information and
practices. Additionally, by evaluating and reporting on programs, counselors are able to learn about
the success of specific programs as well as areas of improvement. By reporting on the outcomes,
counselors are educating the community about the school and its counseling programs. Giving
presentations at professional conferences allows the counselor the opportunity to share different
counseling practices and strategies with other professionals. The sharing of information promotes
professional development and is highly recommended in the counseling profession.

How well did you know this?
1
Not at all
2
3
4
5
Perfectly
4
Q

Mrs. Mason is a high school counselor who has been working with a 9th-grade student in
individual counseling for a month. During their first three sessions together, it becomes clear that the student has significant depression and has not experienced any improvement during the course of individual counseling thus far. Which of the following is the most
appropriate next step for Mrs. Mason?

a. Sign up for a webinar about depression to learn new strategies to help the student.

b. Call a local mental health facility for a crisis evaluation on the student.

c. Seek professional advice from another school counselor in the district.

d. Work with the student and their parent to refer the child to an outside mental health
provider.

A

d. Work with the student and their parent to refer the child to an outside mental health
provider.

In any circumstance where a student requires long-term individual counseling or has
significant mental health concerns beyond the scope of the school counselor’s training, working
with the parent (and student, if appropriate) to seek an outside mental health referral is appropriate. School counselors should not provide long-term mental health counseling in a school setting. Because nothing indicates a risk of crisis, an immediate crisis evaluation without parental permission would not be appropriate. While webinars for professional development and
consultation with fellow professionals could help the counselor support the student in school, ultimately an outside referral is most appropriate for this situation due to the suspected mental health concern and need for long-term counseling.

How well did you know this?
1
Not at all
2
3
4
5
Perfectly
5
Q

Which of the following topics or tasks would not be addressed in the group selection process for school-age children?

a. Informed consent

b. Voluntary participation

c. Parent permission

d. Allaying anxiety

A

d. Allaying anxiety

Each of the first three answer choices describes a topic or task that would be addressed in the group selection process. Informed consent, parent permission, and voluntary participation would all be established in the individual interview and invitation to the group. Allaying anxiety is typically addressed in the initial group meeting or “forming” stage of the group.

How well did you know this?
1
Not at all
2
3
4
5
Perfectly
6
Q

There is little parent involvement in many schools due to large numbers of working parents. What is one strategy to use to increase participation?

a. Offer incentives for participation

b. Focus on the unemployed parents to work with

c. Suggest holding meetings and events on the weekends

d. Offer opportunities at various times during the week

A

d. Offer opportunities at various times during the week

Oftentimes, schools get into the habit of holding meetings at the same times year after year. In the case of parent-teacher conferences, which are often held during the day, working parents may have to take off work to attend. Additionally, many school council and PTA meetings are held in the evening. While this may be more convenient for working parents, busy families often find it difficult to participate in these events. By offering various times and days of these meeting and events, a school can maximize the participation of the parents.

How well did you know this?
1
Not at all
2
3
4
5
Perfectly
7
Q

A student comes into your office with a referral from her English teacher. This is typically
a straight A student; however, her grades have been slipping lately. She states that she has been “stressed out” but that nothing serious is going on. What issues might you want to ask about in order to determine the reason for her lower grades?

a. Self-esteem issues

b. Problems with a friend or boyfriend

c. Difficulty level of her classes

d. Mental health history

A

b. Problems with a friend or boyfriend

There are a number of factors that can affect students, especially during middle and high school. In this case, it would be wise to discuss problems with friends or a boyfriend with this student. Social problems are often a cause of problems in school, including a marked drop in grades. If this student is having problems with a boyfriend, it may be important to determine the seriousness of the problems, as abusive relationships in schools do occur. Since this student
typically does well in school, the difficulty level of the classes would not be the major concern, unless she recently began new advanced courses. Additionally, there is nothing in this student’s history to indicate that she may have problems with self-esteem or mental health issues.

How well did you know this?
1
Not at all
2
3
4
5
Perfectly
8
Q

Grace is a high school student who asks her school counselor for support in choosing a career path. Using Frank Parsons’ career counseling approach, what would be the first step Grace’s counselor should take in beginning the career counseling process?

a. Having a conversation with Grace about her goals, interests, aspirations, and personality

b. Having Grace complete several tests in order to collect data about her personal traits

c. Having Grace research careers that interest her to identify possibilities

d. Having Grace complete the Self-Directed Search to determine her personality type

A
  1. B: Frank Parsons developed the trait-factor approach to career counseling. The trait-factor
    approach utilizes data from tests to match an individual with one best career. Therefore, a strictly trait-factor approach would begin with testing to determine Grace’s traits. Trait-factor approaches do not rely on objective data from conversations. The Self-Directed Search from answer choice D refers to a tool used in John Holland’s personality theory of career counseling.
How well did you know this?
1
Not at all
2
3
4
5
Perfectly
9
Q

Cultural identity is:

a. Primarily influenced by race and ethnicity

b. Fully developed by early adulthood

c. Stable across changing contexts

d. Dynamic and constantly evolving

A

d. Dynamic and constantly evolving

Cultural identity is dynamic and constantly evolving. Cultural identity is multifaceted and is
impacted by race, ethnicity, gender, religion, spirituality, socioeconomic status, profession, and sexual orientation. One’s cultural identity is a social construct and continues to be formed through
social interactions and is impacted by historical and political contexts.

How well did you know this?
1
Not at all
2
3
4
5
Perfectly
10
Q

A school counselor is part of a school team tasked with developing an intervention plan
for a 7th-grade student with severe social/emotional deficits and behavioral concerns. The
team decides that, in order to understand the student’s behavior, they must conduct an evaluation of the student’s functioning in a variety of settings, including at school, at home with the immediate family, and in the community. The team’s approach to evaluation best
represents which of the following theories?

a. Bronfenbrenner’s ecological systems theory

b. Bowen’s family systems theory

c. Vygotsky’s social development theory

d. Piaget’s cognitive development theory

A

a. Bronfenbrenner’s ecological systems theory

Bronfenbrenner’s ecological systems theory proposed that, as children grow, the various
environments (systems) in which they function interact to influence their development. Therefore, in order to understand a child’s development, all of the systems must be evaluated. Bronfenbrenner proposed the following five ecological systems ranging from the most intimate to the broadest: microsystem, mesosystem, exosystem, macrosystem, chronosystem.

How well did you know this?
1
Not at all
2
3
4
5
Perfectly
11
Q

Adam is a 4th-grade student whose grades have been low for several months. His teacher reports that he seems to have a hard time grasping the content being taught in class and has foundational gaps in content knowledge. The school’s intervention team convenes an initial meeting to discuss a plan of intervention for Adam. Which of the following would NOT be an appropriate stakeholder to invite to this meeting?

a. Adam’s parents

b. The classroom teacher

c. The school psychologist

d. The school counselor

A

c. The school psychologist

In most schools, the school psychologist is an appropriate member of a team when a child has
progressed through the initial stages of intervention and is being considered for a special education
evaluation. In the scenario described with Adam, the team is conducting an initial intervention meeting, so psychological or educational testing would not yet be appropriate and the school psychologist would not yet be involved.

How well did you know this?
1
Not at all
2
3
4
5
Perfectly
12
Q

Which of the following describes a classroom accommodation which is likely to be the most beneficial for a student who has difficulty seeing written content on the board?

a. Preferential seating

b. Extended time

c. Wearing glasses

d. Small group instruction

A

a. Preferential seating

Preferential seating is a classroom accommodation which allows for a child to be seated in an
area of the classroom which best addresses their needs; in this case, preferential seating may
include the child sitting near the board for the best chance at seeing what is written. Extended time
is a classroom accommodation, however, the connection between difficulty seeing what is written
on the board and providing extended time is not clear. Wearing glasses would probably help, but
this does not describe a classroom accommodation as glasses are not provided in the classroom.
Cues or reminders to wear prescription glasses may be a good strategy for the student described,
but only if forgetting to wear glasses is an issue. Finally, small group instruction describes an
intervention or instructional strategy, not a classroom accommodation. In addition, the link between not being able to see the board and small group instruction is not very clear.

How well did you know this?
1
Not at all
2
3
4
5
Perfectly
13
Q

A middle school counselor is planning classroom guidance lessons about career options.
Which of the following would be the best use of technology in order to increase student knowledge of various careers?

a. Using a survey platform to gather pre-lesson data about careers that interest students

b. Having students utilize a search engine to research a career of interest

c. Having students create a presentation about a career of interest

d. Using a career search such as O*NET to connect careers to skills and interests

A

d. Using a career search such as O*NET to connect careers to skills and interests

There are a variety of ways in which a school counselor can incorporate technology into classroom guidance lessons. When making choices about technology use in classroom guidance, it is important to consider the objective of the lesson and how technology supports the objective. In this scenario, the goal was to help students learn about various careers, so a career search engine such
as O*NET would provide a wide variety of information and help students discover careers they may not have previously known about. The other answer choices only help students in reference to the careers they already have knowledge of or interest in, so they do not support the objective of
“increasing student knowledge of various careers.”

How well did you know this?
1
Not at all
2
3
4
5
Perfectly
14
Q

A school counselor is working as part of a team to evaluate a student for possible learning disabilities. Which of the following methods of data collection would NOT be appropriate for gathering background information to support the team in selecting appropriate assessments and evaluating the student?

a. Collecting intervention data

b. Reviewing academic records

c. Collecting teacher surveys

d. Gathering a health history

A

c. Collecting teacher surveys

Evaluating a student for a learning disability requires a review of a variety of existing data
from a variety of sources. In addition, reviewing this data can be useful in selecting appropriate assessments for the student. For example, if a review of the child’s academic record indicates a consistent difficulty in mathematics, then there is a reason to assess the child for a learning
disability in mathematics. The review of existing data must be based on objective information and measurable objectives, so teacher surveys would not be appropriate.

How well did you know this?
1
Not at all
2
3
4
5
Perfectly
15
Q

Which of the following is true about brief solution-focused therapy?

a. In brief solution-focused therapy, the counselor must understand the client’s problem before beginning to work on solving it.

b. Brief solution-focused therapy is appropriate when the client is unable to articulate their
desired goal.

c. Brief solution-focused therapy requires the client to make major changes in their daily lives.

d. Brief solution-focused therapy does not require the client to have a mental health diagnosis
to begin counseling.

A

d. Brief solution-focused therapy does not require the client to have a mental health diagnosis
to begin counseling.

Brief solution-focused therapy is, as the name implies, focused on solutions to the client’s
problems. Therefore, the deep analysis and exploration of the client’s problems, past history, or
mental health diagnosis are not necessary to begin counseling. In brief solution-focused therapy,
the client is the expert, articulating their own goals and desired outcomes; the counselor is a guide
who helps the client notice changes and improvements toward the desired outcome. Changes do
not need to be major to be significant. The solution-focused counselor is an expert at noticing the
small changes that are making a difference in the client’s goal progress. There is a great deal of
research supporting brief solution-focused counseling in a variety of client scenarios. However, a
client who is unable to articulate their desired goal (such as one who has a severe intellectual disability or one who is experiencing extreme psychosis) would not be a good candidate for brief
solution-focused counseling.

How well did you know this?
1
Not at all
2
3
4
5
Perfectly
16
Q

Joshua is a 5th-grade student who is frequently disruptive and explosive in the
classroom. Schoolwide and classroom-wide interventions, such as positive recognition, class-based rewards, and modifying classroom procedures to reduce opportunities for inappropriate behavior have not resulted in an improvement in Joshua’s behavior. Joshua’s school counselor is working with the administrative team to develop a behavior plan for him. What would be the best method for collecting data to support the development of this plan?

a. Asking Joshua’s teacher to write a narrative about his behavior throughout the school year and the interventions put into place

b. Collecting all of the discipline incident reports generated when Joshua was suspended
throughout the year and looking for patterns

c. Asking Joshua’s parents to complete a behavior checklist, indicating the behaviors they are
most concerned about

d. Conducting a functional behavior assessment to determine the purpose of Joshua’s behavior
and factors contributing to it

A

d. Conducting a functional behavior assessment to determine the purpose of Joshua’s behavior
and factors contributing to it

In this scenario, the student has progressed through schoolwide and classroom-wide interventions, and targeted individual intervention is appropriate. A functional behavior
assessment (FBA) is a detailed method of collecting data about a student’s behavior in order to
develop an appropriate plan for changing inappropriate behaviors. While anecdotal evidence and
historical data can be helpful, the FBA will provide the most detailed information to create an
effective plan.

How well did you know this?
1
Not at all
2
3
4
5
Perfectly
17
Q

What is the benefit of continued assessments?

a. Student progress is monitored.

b. Areas of weakness are identified.

c. Learning styles and goals can be modified.

d. All of the above

A

d. All of the above

Continued monitoring and assessment of the learning environment can be beneficial in many ways. This process provides the counselor, teacher, student, and parents an ongoing progress
report. Additionally, regular assessments allow problems to be identified sooner rather than later.
This will ensure that the student is not falling behind his classmates or his intervention goals. Ongoing monitoring and assessment also allow the counselor to update and to modify goals and target dates. When considered a fluid process, assessments, goal monitoring, intervention modification, and encouragement will ensure that students are progressing at a comfortable and successful rate.

How well did you know this?
1
Not at all
2
3
4
5
Perfectly
18
Q

If a school counselor has a private practice, why would she refer a student in need of intensive counseling to someone else rather than seeing the client at her private practice?

a. She cannot see a client more than once a week.

b. She does not offer the counseling that this student needs.

c. She must have a full case load with her private practice.

d. She cannot use her position within the school to benefit her private practice.

A

d. She cannot use her position within the school to benefit her private practice.

It is considered an unethical professional practice for a counselor to use the school as a place
to gain private practice clients. While many issues can be resolved within the capacity of a school
counseling program, there may be times when a counselor feels a student needs additional therapy
or support. In these cases, the counselor should discuss these options with the student and the
parents while keeping in mind rules of confidentiality. Additionally, after a referral is made, the school counselor does not have a right to information that the new counselor and student share. In certain circumstances, the school counselor may be asked to consult on various issues. If this
occurs, the school counselor and the private practice professional must discuss the case only in
factual, objective ways.

How well did you know this?
1
Not at all
2
3
4
5
Perfectly
19
Q

A new school counselor is working with his administrator to finalize the comprehensive school counseling program for the upcoming school year. The administrator has asked him to identify the components of the counseling program in terms of tiered interventions. Which of the following activities would best fit tier I of a tiered intervention model?

a. Classroom guidance curriculum

b. Small group counseling

c. Individual counseling

d. Closing-the-gap action plan

A
  1. A: A tiered intervention model layers interventions from tier I (broad, whole-school or whole-
    class interventions), to tier II (more intensive, small-group interventions) and tier III (highly individualized, intensive interventions). Tier I can also be thought of as high-quality instructional practices provided to all students, while tiers II and III are layered on top of tier I. Because the
    classroom guidance curriculum is high-quality instruction provided to all students, it would be considered a tier I intervention. Small group counseling could be considered tier II and individual counseling could be considered tier III in terms of the levels of individual attention and intensity of interventions described by the tiered intervention model. A closing-the-gap action plan could
    certainly be part of a school’s intervention planning, but it is not an intervention in and of itself; the
    action plan would describe interventions to be implemented for at-risk groups of students.
How well did you know this?
1
Not at all
2
3
4
5
Perfectly
20
Q

On which of the components of a comprehensive counseling and guidance program should a counselor ideally spend the majority of their time?

a. Foundation

b. Management

c. Delivery

d. Accountability

A
  1. C: The Delivery component of the comprehensive counseling program includes all of the types of direct student services, which ideally make up 80% of a counselor’s time. The Delivery component includes the comprehensive guidance program, individual student planning, and responsive services; these services are delivered through individual counseling, small group, and classroom guidance formats.
How well did you know this?
1
Not at all
2
3
4
5
Perfectly
21
Q

Mrs. Harris, a school counselor, is approached by a teacher at her school who asks her if she has a moment to talk. The teacher then shares with Mrs. Harris that she is having conflict in her family and is struggling mentally and emotionally with her situation. She asks Mrs.
Harris if she can provide individual counseling to help her through this difficult time. What would be an appropriate next step for Mrs. Harris?

a. Mrs. Harris should listen empathetically but ultimately provide the teacher with a list of
referral resources to find a counselor outside of school.

b. Mrs. Harris should listen and set up regular times to meet with the teacher to talk because
the teacher’s mental health impacts students.

c. Mrs. Harris should steer the conversation away from the teacher’s personal problems and
should only discuss academic concerns with her.

d. Mrs. Harris should discuss the teacher’s situation with HR because they can provide
information about the school’s mental health insurance benefits.

A

a. Mrs. Harris should listen empathetically but ultimately provide the teacher with a list of
referral resources to find a counselor outside of school.

The ASCA Ethical Standards are clear when it comes to dual relationships. In this case, by
providing individual counseling to a teacher, Mrs. Harris puts herself in a situation where she is
creating a dual relationship that could negatively impact students. For example, if a student comes
in later to discuss the problems they are having in that teacher’s class, it would be difficult for Mrs.
Harris to remain neutral because she has been providing individual counseling regarding personal
concerns with the same teacher. The teacher’s mental health indeed impacts students, but providing mental health counseling to the teacher is beyond the scope of the school counselor’s role. Of course, Mrs. Harris should still maintain confidentiality regarding the teacher’s concerns and should not approach HR on behalf of the teacher.

How well did you know this?
1
Not at all
2
3
4
5
Perfectly
22
Q

A counselor has developed a comprehensive developmental counseling program in a
large school. This program consists of daily meetings and programs to address the various needs of students. In order to avoid burnout, how might the counselor effectively manage so many meetings and programs?

a. Schedule meetings a few hours after school ends so she or he can take a break

b. Recruit the help of teachers and peer mentors to assist with the programs

c. Try to combine groups so that there are not so many

d. Rely on consultants to run the programs that are scheduled for after school

A

b. Recruit the help of teachers and peer mentors to assist with the programs

Proper delegation of responsibilities will lead to more effective counseling programs. There
are many opportunities for teachers and students to participate in leading programs. For example,
developing a teacher advisor program increases participation from teachers and offers students a
larger support system. Peer mentoring programs allow mentors the ability to increase their
leadership skills and allow mentees the opportunity to widen their network of support systems.
Counselors who encourage the assistance of teachers and students with various programs will
foster a cohesive school that works together toward a common goal. This cohesiveness will provide
all students with the best possible learning environment.

How well did you know this?
1
Not at all
2
3
4
5
Perfectly
23
Q

Researchers examining teen dating and relationships consider peer influences, personal
beliefs, and environmental factors. This is known as:

a. The zone of proximal development

b. The pleasure principle

c. Reciprocal determinism

d. Psychological hedonism
Refer to the following for question

A

c. Reciprocal determinism

The influence of peer relationships, personal beliefs, and environmental factors is known as
reciprocal determinism. Reciprocal determinism is the foundational concept of Bandura’s social
learning theory, or social cognitive theory (SCT). SCT has been used along with attachment theory,
interpersonal theory, and feminist theory to study factors related to adolescent dating and
relationships. Lev Semyonovich Vygotsky used the term zone of proximal development to describe
the difference between what a child does independently and what they can do with help. This
concept is used to explore ways to teach reading, writing, and other language skills. Sigmund Freud
used the term pleasure principle when explaining the role of the id in personality development.
Psychological hedonism states that all human behavior is dictated by actions directed toward
attaining pleasure or avoiding pain.

How well did you know this?
1
Not at all
2
3
4
5
Perfectly
24
Q

You work in an inner-city school where students are subject to a number of risk factors. The
rate of violent incidences is above average, as are the numbers of drug offensives and teen pregnancies. The school currently has various in-house prevention and intervention
programs offered on a volunteer or referral basis to all students. However, even the most successful of these programs yield few positive results. As the counselor, you decide to restructure the programs in an attempt to increase their effectiveness by collaborating with
the community.
Question: It comes to your attention that some of the teachers in this school are treating students in drug intervention programs differently. As the school counselor, what should you do to stop this behavior?

a. Conduct training for the teachers on discrimination and diversity in the schools

b. Report these teachers to the proper superiors

c. Nothing, your job is to deal only with the students

d. Speak to each teacher individually about the effects of their negativity

A

a. Conduct training for the teachers on discrimination and diversity in the schools

Students in drug intervention programs may be experience a number of feelings and barriers
as they attempt to stop using drugs. Teachers must be aware of these issues, along with the
recovery process. It will be important for those students in intervention programs to receive
support from all staff members to ensure success. As a counselor, you may offer training for
teachers and other staff members. This training can include information about the specific intervention programs offered at your school, the recovery process, and sensitivity training specific to this population of students.

How well did you know this?
1
Not at all
2
3
4
5
Perfectly
25
Q

Self-control procedures include all of the following EXCEPT:

a. Self-monitoring

b. Self-punishment

c. Response control

d. Stimulus control

A

c. Response control

Self-control procedures do not include response control. Self-control procedures include self-
monitoring, self-punishment, stimulus control, and self-reinforcement. Counselors using self-
monitoring encourage clients to record information about the frequency and duration of specified
symptoms. Self-punishment and self-reinforcement help clients self-administer the consequences of
the targeted behavior, so that it can be modified. Stimulus control includes behavior that is
controlled by associated consequences. Stimulus control techniques include narrowing, cue
strengthening, and fading.

How well did you know this?
1
Not at all
2
3
4
5
Perfectly
26
Q

A school counselor would like to observe the social skills of a newly referred third grader. Prior to the intake, the counselor observes the student during physical education
and later during lunchtime. This type of observation is advantageous over a formal observation because:

a. There is a decreased likelihood for the Hawthorne effect.

b. There is a decreased likelihood for confirmation bias.

c. Interrater reliability values are higher.

d. Threats to internal validity are minimized.

A

a. There is a decreased likelihood for the Hawthorne effect.

This type of observation (i.e., an informal observation) is advantageous over a formal
observation because it decreases the likelihood of the Hawthorne effect. The Hawthorne effect
presumes that a subject is more likely to modify their behavior based on knowing that they are
being observed. Because the student was not a current client, the student lacked an awareness of
being observed. Confirmation bias occurs when a counselor seeks information to confirm an initial
hypothesis despite encountering information that may refute that hypothesis. Unstructured
observations are at greater risk for confirmation bias than formal observations. Interrater
reliability values are likely to be lower rather than higher. Likewise, threats to internal validity are
not minimized.

How well did you know this?
1
Not at all
2
3
4
5
Perfectly
27
Q

A student’s standardized score report for academic assessment indicates that their stanine score is 8. Which of the following is a correct interpretation of this information?

a. The student’s score is far below average.

b. The student’s score is below average.

c. The student’s score is average.

d. The student’s score is above average.

A

d. The student’s score is above average.

Stanine measurements divide a standard bell curve into nine equal groups, and a stanine
score places an individual score within one of those nine groups. A stanine score of 5 is the mean, or
average, with stanine 1 being far below average and stanine 9 being far above average. Stanine
scores allow an individual to interpret their score in relation to the rest of the group.

How well did you know this?
1
Not at all
2
3
4
5
Perfectly
28
Q

Mr. Dean, a school counseling student in a graduate program, is applying for internship positions for the upcoming school year and is required to carry professional liability insurance. What is the best method for Mr. Dean to acquire a complete professional liability
policy?

a. Contact his university field experiences office

b. Contact the local school district to be added to their district policy

c. Join the National Board for Certified Counselors

d. Join ASCA as a student member

A

d. Join ASCA as a student member

One major benefit of ASCA membership, even for school counseling students, is the
availability of professional liability insurance. University field experiences offices or local school
districts may add interns to their policies, however, this varies widely. ASCA is the best option for
Mr. Dean. The National Board for Certified Counselors is not an option for school counseling students who have not yet earned their degree.

How well did you know this?
1
Not at all
2
3
4
5
Perfectly
29
Q

A school counselor presents data about their school counseling program to their principal. Which of the following would the counselor be most likely to advocate for using
process data from the previous school year to support their requests?

a. An appropriate counselor-to-student ratio in accordance with ASCA recommendations

b. Additional professional development about working with students with anxiety

c. The reassignment of inappropriate duties to other professionals within the school

d. Time during the year to train teachers and other staff on student mental health

A

a. An appropriate counselor-to-student ratio in accordance with ASCA recommendations

Process data describes the school counseling program in terms of who was served and how. In other words, process data tells when and how often programming occurred and how many students participated. This type of data would be supportive for a counselor advocating for an
appropriate student-to-counselor ratio, especially if their process data demonstrates that they are unable to reach all of the students on their caseload equally or their caseload is much larger than the recommended 250:1. Outcome and perception data is more likely to be utilized to support a request for professional development for counselors or teachers. Other types of data, such as the use-of-time assessment or published research on the training and impact of school counselors,
could be used to advocate for reassignment of inappropriate duties.

How well did you know this?
1
Not at all
2
3
4
5
Perfectly
30
Q

Frank Parsons, “The Father of Guidance,” is best known for his contributions to the school counseling profession focusing on which counseling role?

a. Therapeutic counseling

b. Vocational counseling

c. Promoting systemic change

d. Advocating for the profession

A

b. Vocational counseling

Frank Parsons was the author Choosing a Vocation and the leader of the vocational
counseling movement. A focus on the role of the counselor in a therapeutic role did not occur until
later, with Carl Rogers’ contributions to the profession. Systemic change and advocacy roles came
much later with the introduction of ASCA.

How well did you know this?
1
Not at all
2
3
4
5
Perfectly
31
Q

A high school student experiences the death of their sibling a few days before they are
scheduled to take the SAT. If the student takes the test as scheduled, which of the following is true?

a. Their test score will not be affected since the SAT is a validated assessment.

b. Their test score may not be reliable due to their emotional state.

c. Their test score may not be valid due to their emotional state.

d. Their test score will not be affected since the SAT does not measure social-emotional skills.

A

b. Their test score may not be reliable due to their emotional state.

Test reliability indicates how consistent a test is. Test validity means that the assessment
measures what it is intended to measure. The SAT is a valid and reliable assessment, meaning that
it measures what it is intended to measure and individual test-takers should receive the same or
similar scores on repeated administrations. Individual circumstances, however, such as a recent death in the family, may mean that an individual’s test score is not reliable for that administration.
If that student were to repeat the test at a later date when their emotional state was not as heightened, they may perform differently.

How well did you know this?
1
Not at all
2
3
4
5
Perfectly
32
Q

Ms. White, a high school counselor, is planning a small group focused on improving study
skills. As part of the group, she plans to have students complete an electronic survey describing their independent use of various study skills in both the first and last meetings of the group. Which type of data is Ms. White collecting with this survey?

a. Process data

b. Perception data

c. Outcome data

d. Quantitative data

A

b. Perception data

Pre- and post-test data is a type of perception data that is used to measure gains in knowledge or changes in perspective. Perception data measures what people think they know, believe, or can do. Because of their subjective nature, surveys are commonly used to collect
perception data, but not process or outcome data. Process data measures what a school counselor
does and for whom, so subjective measures such as a survey are not applicable. Outcome data measures the impact of school counseling activities and is much more objective in nature, so this information is typically pulled from the SIS.

How well did you know this?
1
Not at all
2
3
4
5
Perfectly
33
Q

Which of the following is NOT true about most commonly-used intelligence tests, such as
the Stanford-Binet or Wechsler?

a. Intelligence test scores are measured based on a standard bell curve.

b. The average intelligence score is 100, based on the individual’s age group.

c. Intelligence tests are statistically valid and reliable.

d. Intelligence tests are valid across different cultures.

A

d. Intelligence tests are valid across different cultures.

Commonly used intelligence tests are frequently criticized for having cultural biases which impact an individual’s outcome; in addition, intelligence tests are known to measure a very limited type of intelligence. Even with these limitations, intelligence tests can be a helpful tool when utilized as a part of a broader assessment of an individual. For example, when evaluating students for special education services, a child’s score on the Wechsler Intelligence Scale for Children may indicate relative strengths and weaknesses in different measurements of intelligence, which ca help pinpoint specific learning disabilities. It is important, however, to keep in mind the criticism of these tests for being culturally biased and understand that they may not measure intelligence in the same way across different cultures. Like most assessments, intelligence tests should not be used as a single data point for decision making.

How well did you know this?
1
Not at all
2
3
4
5
Perfectly
34
Q

Which stage of the group counseling process represents the majority of the group’s time?

a. Initial/forming

b. Transition/storming

c. Working/norming

d. Termination/adjourning

A

c. Working/norming

The working or “norming” stage of the group counseling process is where the majority of the group’s time is spent, usually about 50% of the time. This is where the majority of the group goals are addressed and the true “work” of group counseling is accomplished. The initial or “forming”
stage is when the group first comes together, goals and group rules are established, and the facilitator tries to alleviate members’ anxiety about the group counseling process. The transition or “storming” stage is when the group begins to transition to the actual work of the group counseling
process. This stage is characterized by conflict and group members challenging the facilitator. If
conflicts are not resolved in this stage, the result can be difficult for the remainder of the group
counseling session. Finally, termination or “adjourning” is the closing of the group, sometimes
characterized by the grief of group members, and is focused on transferring new skills or
experiences to the outside world.

How well did you know this?
1
Not at all
2
3
4
5
Perfectly
35
Q

What is one of the primary reasons why counselors have a difficult time getting support
from immigrant parents?

a. Immigrant parents do not challenge school authority.

b. Immigrant parents are oftentimes unfamiliar with American school systems.

c. Immigrant parents do not have time to talk with school counselors.

d. Immigrant parents have to work during the day.

A

b. Immigrant parents are oftentimes unfamiliar with American school systems.

Immigrant parents have many challenges when sending their children to school in America.
Oftentimes, the parents do not speak English and are unfamiliar with the American school system.
While language barriers often result in parents not participating in their children’s education, a counselor can easily offer translators and other educational opportunities to overcome these barriers. However, if parents continue to be unclear about the needs and expectations of the school system, immigrant parents will continue to be inactive. This can be overcome by providing a supportive environment for both the parents and the students. Counselors in school systems with a high immigrant population will need to serve as “school-home-community liaisons” in order to promote the needs of the students and their families.

How well did you know this?
1
Not at all
2
3
4
5
Perfectly
36
Q

A school counselor is conducting a small group focused on developing social skills. During a group session, a student begins discussing the misbehaviors of another classmate
who is not in the group, making many comments about the student’s lack of social skills. Which technique would be appropriate to use in response to these comments?

a. Clarification

b. Blocking

c. Linking

d. Active listening

A

b. Blocking

In this situation, the counselor’s goal should be to stop the inappropriate conversation. Gossiping about others who are not in the group is not helpful to the group or appropriate in the context of the group. Blocking is a strategy for stopping a hurtful behavior. Clarification, linking, and
active listening would all be inappropriate choices in this scenario because they would invite the
student to continue their line of discussion.

How well did you know this?
1
Not at all
2
3
4
5
Perfectly
37
Q

A state requires all of its students to take an assessment each year that measures mastery of grade-level content standards taught during the year. This type of assessment is best described as which of the following?

a. Achievement test

b. Aptitude test

c. Placement test

d. Norm-referenced test

A

a. Achievement test

Achievement tests measure how well the test-taker has mastered specific skills or objectives.
End-of-grade or end-of-course tests, as required by many states and school districts, are examples
of achievement tests because they measure a student’s mastery of the content objectives or skills
that are generally expected to be taught during that grade level or course. While some schools may
use data from these achievement tests to make placement decisions for future courses, that is not
what these types of tests are designed for. Aptitude tests are a different type of assessment that
attempts to predict the test-taker’s aptitude or future success in a specific area, while placement
tests measure the test-taker’s skill level in a specific content area in order to determine their current level for course placements. Norm-referenced tests measure the individual test-taker’s or group of test-takers’ results against another group of test-takers, while criterion-referenced tests measure the individual test-taker’s results against a specific set of standards. For example, a norm-referenced test may indicate that a student’s mathematics ability is more or less advanced than a group of their same-aged peers, while a criterion-referenced test would simply indicate that a student has mastered all of their current grade-level mathematics standards.

How well did you know this?
1
Not at all
2
3
4
5
Perfectly
38
Q

When dealing with middle school students, it is important for a counselor to understand
the developmental relationship between Industry and Inferiority. This is an example of a theory of development based on the work of what psychologist?

a. Freud

b. Piaget

c. Erikson

d. Bandura

A

c. Erikson

Erikson suggests that children begin to associate with either industry or inferiority in early
adolescence. Industry refers to a person’s ability to succeed and feel worthy. Typically, healthy children who receive the appropriate support and encouragement at home and at school will feel a sense of industry. This identification is important, as it can be the basis of future educational, social, and coping skills. For example, a child who has a strong sense of industry will be more likely to cope with a low grade on a test. He will understand that he may need to study harder for the next test or seek assistance. On the other hand, a child with a poor school, family, or peer support system may experience feelings of inferiority. In this case, he is likely to have insufficient coping mechanisms
and not perform well in school.

How well did you know this?
1
Not at all
2
3
4
5
Perfectly
39
Q

An elementary school has two school counselors. They each maintain their own caseload
for classroom guidance, individual counseling, and small groups. Each year, the two counselors plan and implement a schoolwide career day, sharing equally in the planning and
other tasks. Which of the following school counselor roles are the counselors demonstrating in the planning of career day?

a. Consulting

b. Collaborating

c. Evaluating

d. Counseling

A

b. Collaborating

This example describes collaboration and could also be considered an example of “leaderless” collaboration, where all parties in the collaborative effort have equal leadership status.
Co-counselors within the same school may also conduct each of the other roles listed independently
or together. For example, counselors may frequently consult with one another on particularly difficult cases, ethical dilemmas, or areas of individual expertise. Effective counselors will also likely conduct an evaluation of their programming together. Although likely less common in a school setting due to availability of time, counselors may also at times conduct counseling together, particularly in groups.

How well did you know this?
1
Not at all
2
3
4
5
Perfectly
40
Q

A school counselor conducts a needs assessment with students and staff. Using the data gathered from the needs assessment, the counselor could do all of the following EXCEPT:

a. Identify small-group counseling topics

b. Design classroom guidance curriculum

c. Identify individual student needs

d. Evaluate the previous year’s program

A

d. Evaluate the previous year’s program

A well-designed needs assessment can be a powerful tool for school counselors in identifying
areas of need, defining goals, and planning programming. Needs assessments can also identify areas
where there may be equity gaps, individual student concerns, or common student concerns that
may be addressed by small-group counseling. While it may be tempting to use a needs assessment to track changes in data from year to year as a form of program evaluation, this is not the purpose for which they are designed and are unlikely to be effective for program evaluation due to changes in student enrollment or staffing from year to year and a lack of specificity within the questions to
evaluate individual programs.

How well did you know this?
1
Not at all
2
3
4
5
Perfectly
41
Q

An educational trend that directly impacts the role of school counselors is the multi-tiered systems of support (MTSS) framework. Which of the following describes an appropriate role for school counselors in implementing MTSS?

a. Coordinating referrals for students into the intervention process as well as maintaining
appropriate data collection

b. Coordinating standardized testing programs in order to help identify appropriate interventions

c. Delivering standards-based academic, social/emotional, and career curriculum to all students

d. Providing individual counseling to all students identified as tier III students

A

c. Delivering standards-based academic, social/emotional, and career curriculum to all students

School counselors support the implementation of MTSS by providing a standards-based
curriculum to all students (the intervener role). It is not appropriate for school counselors to lead
the implementation of MTSS as a whole because MTSS is a team approach. Even within an MTSS
framework, it is still inappropriate for a school counselor to coordinate a testing program or to
provide standard individual counseling to all students in a certain tier of MTSS.

How well did you know this?
1
Not at all
2
3
4
5
Perfectly
42
Q

A school counselor is tracking graduation rates over the course of several years to determine if a program created to increase the four-year graduation rate is creating the desired result. In analyzing the data, the school counselor is specifically examining which of
the following?

a. Trend

b. Mean

c. Mode

d. Standard deviation

A

a. Trend

A statistical trend is a method of measuring a change over time. There are a variety of methods and tests which can be utilized to analyze statistical trends. If a statistically significant trend exists within a set of data (such as graduation rates over the course of multiple years), then
that trend could be understood as the overall direction of data. In the example described, there could be a trend toward increasing or decreasing graduation rates. It is also possible to have no trend within a data set. The mean is the average of the data, which is not helpful in analyzing changes over time in this example. The mode is the most common value within the data, and the standard deviation has to do with distribution along a bell curve, neither of which are helpful pieces
of information for the example given.

How well did you know this?
1
Not at all
2
3
4
5
Perfectly
43
Q

Elementary school counselors must have a strong understanding of what?

a. Child development

b. Continued education options

c. Relationship building

d. Adjustment and coping mechanisms

A

a. Child development

Elementary School is the time when children are learning the most and developing important
motor skills and mental functions. Counselors must be able to recognize developmental delays, as
well as advanced learners. School counselors will be major advocates for students who may experience developmental delays. While the other options may be important, these issues are more likely seen at the junior high and high school levels. Knowledge in continued education will be important for high school students who wish to pursue a college education; counselors with
students in these age groups will need to educate students and offer assistance in applying to colleges and obtaining financial aid and scholarships. Additionally, relationship building, adjustment, and coping mechanisms are issues most often seen in junior high school, as these years are a time of many changes physically, emotionally, and educationally.

How well did you know this?
1
Not at all
2
3
4
5
Perfectly
44
Q

A school counselor has several students with anxiety disorders. The counselor would like to provide support for those students in school and is considering small-group counseling.
Which of the following would NOT be a benefit to utilizing small group counseling for this purpose?

a. Meeting with more students at one time

b. Opportunities for peer modeling

c. Lower anxiety levels in a group setting

d. Opportunities for social exposure

A

c. Lower anxiety levels in a group setting

Small group counseling can be very beneficial for students with various anxiety disorders,
and there are multiple programs and curricula available for counselors to use for this purpose.
There are several benefits for both the students and the counselor in utilizing small group
counseling for this purpose. A busy school counselor with a large caseload can see more students at
one time in a small group setting, potentially increasing their effectiveness in reaching all students.
Students in a group also benefit from peer exposure, learning from others’ experiences, practicing
new skills together, and having a level of social exposure that can help build confidence in social situations. However, group members should be carefully considered because not all students with anxiety can immediately function in a small group setting. Some students with severe social anxiety may need preparatory individual counseling to be able to benefit from small group counseling.
Therefore, answer choice C is correct because not all students are less likely to be anxious in a group setting.

How well did you know this?
1
Not at all
2
3
4
5
Perfectly
45
Q

Which major component of a comprehensive school counseling program most directly allows a school counselor to demonstrate school-level leadership?

a. Counseling curriculum

b. Individual student planning

c. Responsive services

d. System support

A

d. System support

The system support component of the comprehensive school counseling program utilizes a counselor’s leadership skills to support the total school environment and promote systemic change. System support activities such as professional development, committee work, and program management all provide opportunities for school counselors to demonstrate leadership and
advocacy at the school level. While there could be some components of counseling curriculum, individual student planning, and responsive services that allow a counselor to demonstrate leadership, the system support or indirect services component is more strongly related to leadership roles.

How well did you know this?
1
Not at all
2
3
4
5
Perfectly
46
Q

The counseling theory focusing on feelings of inferiority vs. superiority and the birth order of the client is based on whose work?

a. Erik Erikson

b. Alfred Adler

c. Sigmund Freud

d. Erich Fromm

A

b. Alfred Adler

Alfred Adler’s theory of individual psychology focused on feelings of inferiority vs. superiority and Adlerian therapy investigates the effect of the client’s birth order on behavior. Adler
was once a colleague of Sigmund Freud, but he eventually separated from Freud’s circles.

How well did you know this?
1
Not at all
2
3
4
5
Perfectly
47
Q

Which of the following documents provides an outline of objectives that can be used in
planning a classroom guidance curriculum?

a. ASCA Ethical Standards for School Counselors

b. ASCA Mindsets & Behaviors for Student Success

c. ASCA National Model

d. Recognized ASCA Model Program

A

b. ASCA Mindsets & Behaviors for Student Success

The ASCA Mindsets & Behaviors for Student Success outline skills and mindsets essential for
student success in college and beyond. Counselors planning a classroom guidance curriculum
should utilize the Mindsets & Behaviors to begin to outline learning objectives for their lessons.

How well did you know this?
1
Not at all
2
3
4
5
Perfectly
48
Q

Ms. Howell is a first-year counselor at an elementary school. In preparation for the upcoming school year, she works with her building principal to draft mission and vision statements for the counseling program. Writing a mission and vision statement most closely aligns with which component of the ASCA National Model?

a. Define

b. Manage

c. Deliver

d. Assess

A

b. Manage

Development of the mission and vision statements for the counseling program establish
program focus and define student outcomes, which are part of the Manage component of the ASCA

How well did you know this?
1
Not at all
2
3
4
5
Perfectly
49
Q

Counseling program evaluations serve a variety of purposes for both the school counseling program and the profession as a whole. Which of the following is NOT a purpose
served by school counseling program evaluations?

a. Sharing results with stakeholders

b. Advocating for the needs of the counseling program

c. Identifying future program goals

d. Advocating for a pay raise

A

d. Advocating for a pay raise

The program evaluation is designed to serve each of the listed purposes except for advocating for a pay raise. While school counselors advocate in general for the profession, and that
may include advocating for adequate professional compensation, the individual program evaluation
is not designed to serve this purpose.

50
Q

An individual has begun to compare their accomplishments to those of their peers and feels a sense of pride in how they compare to others. According to Erik Erikson, this individual is in which stage of development?

a. Autonomy vs. shame (ages 2-3)

b. Initiative vs. guilt (ages 3-6)

c. Industry vs. inferiority (ages 6-12)

d. Identity vs. role confusion (ages 12-18)

A

c. Industry vs. inferiority (ages 6-12)

In the industry vs. inferiority stage of development, an individual begins to compare their personal accomplishments (academic, relational, athletic, etc.) to their peers and as a result, develops either a sense of pride or a sense of inferiority in comparison to others. The autonomy vs. shame stage focuses on a developing the senses of independence and autonomy in a child who is given the opportunity to make basic decisions about their lives, or a sense of shame when the child is not given such input. In the initiative vs. guilt stage, a child learns a sense of pride when allowed to work toward a goal and practice responsibility, or a sense of guilt when they are unsuccessful in
this task. Identity vs. role confusion is the stage in which adolescents grapple with the question,
“Who am I?”.

51
Q

Which of the following is NOT an example of a criterion-referenced assessment?

a. End-of-grade exams

b. End-of-unit assessments

c. AP exams

d. The SAT

A

d. The SAT

Criterion-referenced assessments measure the test-taker’s mastery of specific skills or
objectives; results can typically be expressed in terms such as “passing” or “proficient.” Students
taking criterion-referenced assessments have either mastered the objectives or not; the abilities of
other students taking the assessment have no impact on their score or test result. Norm-referenced
assessments, however, compare the individual test-taker to their peers. These results are typically
expressed as a percentile rank, rather than “passing” or “proficient.”

52
Q

Which of the following school counselor roles is considered a direct service?

a. Consulting

b. Collaborating

c. Evaluating

d. Counseling

A

d. Counseling

Counseling is a direct student service, while consulting, collaborating, and evaluating are all
indirect services. Specifically, counseling is a responsive, direct service. Consultation and
collaboration can be easily confused for direct services because they involve working directly with
other professionals or stakeholders, often in “direct” response to a student need. However, direct
services are defined as services provided to students.

53
Q

In which of the following scenarios might a personality assessment, such as the Myers-Briggs Type Indicator (MBTI), be an appropriate tool?

a. Determining placement in honors coursework

b. Evaluating students for special education services

c. Identifying careers that may be of interest

d. Diagnosing an individual with a personality disorder

A

c. Identifying careers that may be of interest

Personality assessments, such as the Myers-Briggs Type Indicator, can be useful tools for a
variety of purposes, including self-reflection, learning coping skills, and identifying areas of
academic or career interest. They are not, however, diagnostic assessments to be used for
diagnosing personality disorders, evaluating for learning disabilities, or determining placement in
special education or honors coursework. Personality assessments like the MBTI require an individual to be able to accurately reflect on and interpret their own thoughts, feelings, and
behaviors and rate them accurately and therefore should never be utilized as a sole data point in
decision making.

54
Q

Alex is a middle school student who struggles with focus and attention in class. Alex’s difficulties frequently result in incomplete work and poor grades on assessments. Alex’s 504
team determined that he would benefit from preferential seating and extended time on assignments. Which of the following terms best describes preferential seating and extended time?

a. Intervention

b. Accommodation

c. Modification

d. Instruction

A

b. Accommodation

When it comes to providing equal access to the curriculum via a Section 504 plan or IEP, the most common terms used are “accommodation” and “modification.” While counselors will
frequently hear and use these terms in conjunction with each other, they refer to different types of strategies. Accommodations are those strategies that change the environment, time, type of output, or size of the task at hand without fundamentally changing the task, instructional level, or content that the student is learning. Accommodations allow students to access the curriculum as well as their non-disabled peers and perform the same learning tasks as their peers. Preferential seating
(such as seating away from distractions or close to the teacher) is an example of environmental
accommodation. Extended time is an accommodation that changes the length of time given to complete a task without changing the task itself. Modifications, however, change the task or content itself based on a child’s needs. Modifications may lower or change the learning standards expected of the student. An example of a modification would be giving a child an alternate reading passage on their reading level. Interventions are instructional strategies used to help a child learn content; an example would be small group instruction.

55
Q

Studies have suggested that training in social skills benefits gifted and special needs students. What specific factor does such training address?

a. Self-esteem

b. Sociability

c. Attendance rates

d. Family risk factors

A

b. Sociability

Researchers have studied the effects of providing social skills training to gifted and special
needs students. These trainings often take a cognitive-behavioral approach to learning new behaviors. The results of these studies show that these students demonstrate a significant increase in their sociability. This includes their willingness to participate in social situations, such as on the playground. Social skills trainings often focus on listening skills, empathy, building rapport, self-
disclosure, and appropriate eye contact. This training typically does not affect a student’s self-esteem or other school-related issues, such as attendance or grades.

56
Q

Mrs. Black is holding the first of six group counseling sessions. Which of the following is NOT an appropriate goal for this stage of the group counseling process?

a. Explaining the purpose of the group

b. Establishing behavioral and confidentiality expectations for group members

c. Encouraging the development of group cohesion

d. Addressing conflict between group members

A

d. Addressing conflict between group members

Each of the first three options describes a typical, appropriate goal for the initial or “forming”
stage of group counseling, which typically takes place in the first or second session. Conflict
between group members typically develops in the transition or “storming” stage of the group. Later
stages include working or “norming,” and termination or “adjourning.”

57
Q

Jim, a member of a counseling group focused on overcoming social anxiety, shares his recent success with utilizing skills learned from the group to navigate a tricky social
situation. Jim is excited about his success and how the skills he learned helped him in that
situation; the other members of the group are excited for him as well.

Which benefit of group
counseling would be directly demonstrated by this scenario?

a. Installation of hope

b. Universality

c. Group cohesiveness

d. Imparting information

A

a. Installation of hope

This example most directly demonstrates the idea of “installation of hope,” whereby group
members can see others’ successes and feel hopeful about their own prospects for growth and
improvement. The concept of universality describes the understanding that what group members are experiencing is not unusual and they are not alone in their experiences. Group cohesiveness explains the sense of belonging that group members feel when working toward a common group goal. “Imparting information” could be a tricky answer choice in this scenario. Jim is sharing his
experience with the group, but not necessarily sharing new information that they could learn from.
“Installation of hope” is a better fit for this situation because the information Jim shared could
inspire other group members in their progress.

58
Q

Sarah is a 4th-grade student who, for the past seven months, has demonstrated restlessness and an inability to focus at school. She is frequently irritable and snaps at her
classmates, which has resulted in difficulty maintaining friendships. She complains of frequent headaches and stomach aches and often seems tired. Sarah’s mother shared with the school counselor that Sarah has difficulty sleeping at night and there is no known
medical cause for her headaches or stomach aches. Which of the following disorders could
Sarah’s symptoms be consistent with?

a. Generalized anxiety disorder

b. Autism spectrum disorder

c. Oppositional defiant disorder

d. Attention-deficit hyperactivity disorder

A

a. Generalized anxiety disorder

Restlessness, difficulty concentrating, irritability, difficulty sleeping, headaches, and
stomachaches persisting for more than six months are all symptoms of generalized anxiety
disorder. None of the symptoms described are consistent with autism spectrum disorder. Although
irritability and inability to focus could be symptoms of oppositional defiant disorder and attention-deficit hyperactivity disorder, the other symptoms described are not consistent with those
disorders.

59
Q

When would a formative assessment be conducted?

a. Prior to an intervention

b. During an intervention

c. After an intervention

d. Only if the student refuses an intervention

A

b. During an intervention

Formative assessments are used to determine the effectiveness of an intervention and can be
used to modify the treatment. This is an important step in the intervention process, as a student’s
rate of learning is not always apparent. This type of assessment can be in the form of informal
questions to determine if the student understands the topic. The goal of a formative assessment is
to provide the best possible setting for the student in order to increase learning and understanding.
When done at regular intervals, the counselor can determine if the goals of the intervention are
being met in a timely manner. A formative assessment will also provide an indication of an
ineffective intervention strategy.

60
Q

Mrs. Jones is a high school counselor working with a student in individual counseling. The student is struggling with math class, and recently received a failing grade on an exam. In the counseling session, the student makes the statement, “I failed my math test. I am so
stupid!” If Mrs. Jones is taking a cognitive behavioral therapy-based approach with this student, which of the following would be an appropriate response to the student’s statement?

a. Challenging the student’s irrational belief that they must be stupid if they received a failing
grade on a test

b. Asking the student about their previous experiences with math tests to explore the root of
the student’s struggles

c. Asking the student if there has been a math test they performed well on and trying to
identify what the student did differently on that test

d. Reflecting the student’s statement, validating their feelings, and demonstrating
unconditional positive regard for the student

A

a. Challenging the student’s irrational belief that they must be stupid if they received a failing
grade on a test

Cognitive behavioral therapy focuses on changing irrational thoughts that lead to unwanted
or negative feelings. In this scenario, the counselor would challenge the student’s belief that they
received a failing grade on their math test because there is something inherently wrong with them
or their intelligence. Instead, the counselor may help the student reframe their difficulty, such as by
saying, “I received a failing grade on my math test because I did not prepare and study effectively.”
Meanwhile, answer choice B would be a problem-focused approach, and answer choice C reflects a
more solution-focused approach. Answer choice D represents a person-centered approach.

61
Q

A counselor who assesses a student according to his or her microsystem, macrosystem, and exosystem will have a complete picture of the student’s cultural and racial identity.
What is this model called?

a. Systems model

b. Ecological model

c. Cultural assessment model

d. Full overview model

A

b. Ecological model

The ecological model consists of the microsystem, macrosystem, and excosystem. The
microsystem encompasses the students’ immediate family and support systems. This will include
parents, siblings, extended family, and friends. The macrosystem is defined by the students’ culture.
This will vary from student to student and often may include various subcultures. The exosystem
refers to students’ social support systems, such as their community. Understanding these three
aspects of students’ support systems will assist in developing appropriate programs and
interventions, as the counselor will be able to incorporate the students’ beliefs, morals, and values
into counseling programs.

62
Q

A counselor is working individually with a student toward their goal of developing friendships and feeling more connected at school. According to the theory of brief solution-
focused therapy, how would the counselor approach termination of the student’s individual counseling sessions?

a. Wait until the student reports that they are at a 10/10 on the scaling question

b. Terminate after six sessions, as anything longer is not “brief”

c. Ask the student how they will know when they no longer need individual counseling

d. Wait until the counselor observes that the student has achieved their goal

A

c. Ask the student how they will know when they no longer need individual counseling

As with most brief solution-focused counseling, the process of termination is similarly guided
by the client’s understanding of their own goal and picturing their desired outcomes. Asking the
client how they will know when they are done with their counseling sessions allows the client to
visualize their desired outcome and articulate the specifics of their goal. While there is not a limit to
the number of sessions in this type of therapy, the number of sessions is typically limited because
clients make progress quickly. Goal progress and readiness for termination are measured by the
client’s perception, not the counselor’s observation. The client may not have completely met their
goal by the time they are ready to terminate their counseling sessions, but rather may have made a
great deal of progress toward that goal and feel confident in their ability to continue to make
progress.

63
Q

Refer to the following for question:
You work in an inner-city school where students are subject to a number of risk factors. The
rate of violent incidences is above average, as are the numbers of drug offensives and teen pregnancies. The school currently has various in-house prevention and intervention
programs offered on a volunteer or referral basis to all students. However, even the most successful of these programs yield few positive results. As the counselor, you decide to restructure the programs in an attempt to increase their effectiveness by collaborating with
the community.

Question: You would like to enhance your school’s drug prevention programs. Which community
agency will you most likely collaborate with?

a. Local police department

b. Local courthouse

c. Local jail

d. Local AA group

A

a. Local police department

Students may benefit from speaking to representatives from all of these organizations.
However, a representative from the local police department would be the best option. These
individuals are usually trained in drug education for school-aged children. Additionally, most police
departments have drug prevention programs in place that have been proven to have positive
outcomes. By bringing in individuals from the community, students get a different perspective of
the issue at hand. Students will be able to ask questions and receive life experience answers from
the presenter. This helps to show students the reality of the problem, as well as its consequences.

64
Q

Which of the following school counselor roles is NOT appropriate in relation to standardized testing?

a. Disaggregating test results in order to drive instructional and programming decisions

b. Providing classroom guidance for students designed to reduce test anxiety

c. Coordinating the school’s testing program, including accommodations for students with disabilities

d. Advocating for standardized tests to be culturally and linguistically unbiased

A

c. Coordinating the school’s testing program, including accommodations for students with disabilities

ASCA does not support school counselors in taking on the role of schoolwide testing coordinator because this task detracts from the role of a counselor in providing academic, social/emotional, and career support to students.

65
Q

A school counselor completes a school data profile in order to plan their school counseling program. In analyzing the data gathered, the counselor notes that the school’s
graduation rate has gradually decreased over the last few years among students with disabilities. Based on the available data, what ASCA National Model document should the counselor use to address this concern?

a. Curriculum action plan

b. Small group action plan

c. Closing-the-gap action plan

d. Lesson plan

A

c. Closing-the-gap action plan

Closing-the-gap action plan documents are used to develop plans to address discrepancies
between groups of students. In the situation described, there appears to be a discrepancy between
students with disabilities and nondisabled students in the rate of high school graduation, so a
closing-the-gap action plan would be appropriate. Within that plan, a counselor may identify activities for classroom guidance lessons or small group counseling, but the overall plan to address the discrepancy is the closing-the-gap action plan. In addition, the documentation of activities and evaluation methods takes place within the closing-the-gap action plan template.

66
Q

A recent transfer student came from an inner-city school to your rural school. She is having difficulty adjusting to the area and the school. As the counselor, what steps can you
take to ease this student’s transition?

a. Announce that there is a new student and ask volunteers to show her around the school

b. Create a support team with the help of other students, teachers, and the new student’s parents

c. Provide Solution-Focused Brief Counseling to the student to find out the underlying problem

d. Provide information about making friends and offer this information to the student and her
parent

A

b. Create a support team with the help of other students, teachers, and the new student’s parents

Making a broad announcement about the student’s arrival is likely to embarrass the student and possibly create more alienation from the general student body. As this student does not necessarily have a problem, direct counseling and education may not beneficial. School counselors
who offer the student, as well as her family, supportive services will be the most beneficial in easing the transition from one school to the next. Counselors can enlist the support and assistance from
other students by providing the new student a peer guide for the first couple of days. The guide will
show them around the school and make sure the student knows where her classes are. This guide
can also provide information about the school, such as the various clubs and sports available and upcoming events. Additional support from the school and the community will provide the student and her family with important information about the school and the new area.

67
Q

A school counselor is leading a small group focused on improving study skills. One group member continually asks other students if they are doing their homework, using their study skills, and focusing in class. This student reminds their peers that they are supposed to be
doing their assignments and studying regularly. Which group role is this student demonstrating?

a. Scapegoat

b. Gatekeeper

c. Energizer

d. Interrogator

A

b. Gatekeeper

The gatekeeper of a group is the member who tries to keep everyone else on task and ensure they are participating in the group. The scapegoat is the group member whom everyone else tries to
blame when things go wrong. The energizer is the group member who attempts to get others excited and enthusiastic about the group, and the interrogator asks a lot of questions of the group.

68
Q

What is one strategy a counselor can use to increase job experience and skills that would require community collaboration?

a. Creating an on-the-job training program

b. Finding part-time jobs for students

c. Offering more career-based or skill-improvement courses, such as auto shop and computer applications

d. Offering programs that focus on resume writing and interviewing skills

A

a. Creating an on-the-job training program

Counselors can be responsible for more than just the mental and emotional well-being of
students. When a school offers various career development programs, students who participate
gain valuable skills for the future. On-the-job training programs are an excellent way to increase job
skills. Additionally, these programs often help students decide what career path to follow. These
programs allow students to go to school and work while earning school credits and gaining practical business experience. Counselors must understand that cooperation and communication with the community and leaders in the business community can be the key to success for these types of programs.

69
Q

A 14-year-old client is involved in a neighborhood gang, is truant, and is using drugs on a daily basis. Which treatment program would be most efficacious for this client?

a. The Incredible Years

b. Parent-implemented intervention

c. Self-help (e.g.,12-step groups)

d. Multisystemic therapy (MST)

A

d. Multisystemic therapy (MST)

MST is an evidence-based practice (EBP) for adolescent substance abuse and associated issues. MST addresses severe behavioral issues, including community influences, school issues, and family factors. Family involvement is a critical element for adolescent substance use disorder treatment. The Incredible Years is an early childhood EBP for children with challenging behaviors, including symptoms of ADHD and ODD. The parent-implemented intervention is an EBP for children with ASD. Finally, self-help groups, including Alcoholics Anonymous and Narcotics
Anonymous, are not EBPs for adolescent substance use and antisocial behaviors.

70
Q

Unrealistic demands on time and job duties refer to what?

a. Role ambiguity

b. Role conflict

c. Role mutations

d. Role confusion

A

b. Role conflict

Role conflict refers to unrealistic demands on the counselor. This often occurs when counselors do not establish appropriate boundaries between themselves and the students. In addition, not establishing boundaries with teachers can lead to unrealistic expectations of the
counselor. Role conflict often leads to counselor burnout because the counselor tries to help everyone. On the other hand, role ambiguity occurs when an individual is unclear about his role within a particular occupation. Role mutations occur when counselors serve in roles not intended for those in the counseling profession, which often leads to inconsistent counseling practices and
programs. Schools and counselors must be clear on the expectations and responsibilities of the school counselor in order to provide effective programs for the students.

71
Q

Mrs. Johnson is a middle school counselor who has been working with a 7th-grade student in individual counseling. The student was diagnosed by an outside therapist with an anxiety disorder and regularly earns failing grades on tests despite excellent classwork and
homework grades. Mrs. Johnson convenes a team meeting to discuss appropriate accommodations for the student under Section 504. Which role of the school counselor was
Mrs. Johnson primarily demonstrating in this situation?

a. Advocate

b. Collaborator

c. Leader

d. Consultant

A

a. Advocate

In this situation, Mrs. Johnson primarily was advocating for her student’s legal rights under
Section 504. Typically, the leadership role applies to the development and delivery of a comprehensive school counseling program, while the consultant role usually involves a third party. Convening a team meeting may involve collaboration, but the counselor’s role in that process was as an advocate for the rights of her student.

72
Q

A school utilizes a reward system in which students can exchange tickets for small prizes. A student at the school has been demonstrating the behavior of blurting out answers instead of raising his hand. When he raises his hand before blurting out an answer, his teacher gives
him a ticket for a prize. According to B.F. Skinner’s operant conditioning theory, the ticket represents which type of operant?

a. Positive reinforcement

b. Negative reinforcement

c. Punishment

d. Neutral operant

A

a. Positive reinforcement

A reinforcement is an operant that increases the likelihood of the target behavior happening
again. Reinforcements can be positive or negative. Positive reinforcements reward the target
behavior with something pleasant or enjoyable, while negative reinforcements reward the target
behavior with the removal of something unpleasant or undesirable. B.F. Skinner’s famous
experiment involved rats who escaped a painful electric current by pushing a lever, which was a
negative reinforcement. Punishment is the opposite of reinforcement as it decreases the likelihood
of the behavior happening again. Neutral operants do not affect the frequency of the target
behavior.

73
Q

What is one strategy that a high school counselor can implement to increase parent participation in their children’s academics?

a. Provide training sessions that teach parents to be tutors for their children

b. Require parents to sign a form confirming that students have completed outside reading
assignments

c. Suggest that parents help students with homework every night

d. Offer regular reports and information on student performance, as well as school events and
programs

A

d. Offer regular reports and information on student performance, as well as school events and
programs

Keeping parents informed about their students’ academic progress is one important factor in
the success of a student. These reports are useful in tracking a student’s progress and identifying
when intervention is necessary. By including the parents in these reports, counselors can take a
step in gaining the parents’ participation and cooperation if problems do arise. These reports also

74
Q

A counselor is researching best practices for working with a family whose children witnessed a violent crime and are now experiencing anxiety, having difficulty focusing,
showing physical illness, and displaying inappropriate behavior. Which of the following research-based approaches would this counselor most likely consider in working with these children?

a. Family system therapy

b. Trauma-informed practices

c. Multi-tiered systems of support

d. Positive behavior intervention and support

A

b. Trauma-informed practices

Trauma-informed practices would be the most reasonable place for this counselor to start
when working with this family. The children experienced a traumatic event and are demonstrating
classic effects of trauma. Family systems therapy would be conducted in a mental health setting
with a trained therapist. Multi-tiered systems of support and positive behavior intervention and
support may be somewhat helpful for the children, but trauma-informed practices are the best
support for recovery from a traumatic experience.

75
Q

A middle school counselor is planning an anti-bullying curriculum for classroom guidance lessons. Which of the following statements is NOT true about middle school
students and bullying?

a. Middle school girls are more likely to experience social bullying than physical bullying.

b. Middle school students are likely to experience or participate in cyberbullying.

c. Middle school students who are “bystanders” are encouraging bullying to continue.

d. Middle school boys do not experience or participate in social bullying, only girls.

A

d. Middle school boys do not experience or participate in social bullying, only girls.

Social bullying is bullying that focuses on damaging or reducing a victim’s social
relationships, including exclusion or spreading rumors. Social bullying is most common in female
students. While middle school boys are more likely to experience or participate in physical bullying,
such as hitting, kicking, or fighting, they are not immune to social bullying. Cyberbullying can be a
form of social or verbal bullying, including name-calling, spreading rumors, making threats,
exclusion, and other types of harassment; cyberbullying is common at the middle school level.
Finally, many bullying prevention programs identify various roles students play in bullying,
including the role of “bystander” or “onlooker,” a student who witnesses bullying but does nothing
to stop the bully or support the victim; a student in this role is considered to be supporting the
bullying behavior.

76
Q

Mr. Lowe, a classroom teacher, meets with the school counselor to discuss a student, Erin. Erin has a diagnosis of generalized anxiety disorder (GAD), and Mr. Lowe is seeking information about how to help mitigate the impact of anxiety on Erin’s classroom behavior.
During their meeting, the counselor shares general information about GAD as well as common strategies and techniques classroom teachers use to support students with this
diagnosis. Their meeting could best be described as which of the following?

a. Consultation

b. Collaboration

c. Professional development

d. Referral

A

a. Consultation

In this situation, the counselor is serving as the “resident expert” to provide information to the classroom teacher, who is seeking information to support his student. They are not working together to come up with a solution, which would be an example of collaboration; it is a one-way
exchange of information. Consultation could be a method by which counselors provide professional development, such as by providing information to a large group of teachers. Although this
consultation could result in an outside referral, as in choice D, the consultation itself is the best
description for the meeting in this scenario.

77
Q

Ms. Joseph has been working with an anxious student individually for several weeks. It is
clear that the child’s needs are significant and the anxiety is interfering with his ability to
attend school. Ms. Joseph recognizes that her training is not adequate for the needs of the student. What is Ms. Joseph’s ethical obligation in this scenario?

a. Ms. Joseph should seek additional training and development in this area in order to better
support the student.

b. Ms. Joseph should refer the child’s parents to a counselor she knows who has been effective with other anxious students in the past.

c. Ms. Joseph should stop seeing the child and tell his parents that he should see an outside counselor.

d. Ms. Joseph should provide the child’s parents with a list of referrals and continue working
with the child in the meantime.

A

d. Ms. Joseph should provide the child’s parents with a list of referrals and continue working
with the child in the meantime

The ASCA Ethical Standards for School Counselors indicate that counselors have a duty to refer when students need a higher level of support than a counselor can adequately provide. In making referrals, counselors should be careful not to endorse any specific provider and, when possible, should provide parents with a vetted list of possible counselors that is approved by the district. The duty to refer does not apply to counselors who feel that they cannot provide services to
a student based on their personal or religious beliefs and, in those scenarios, counselors should
seek additional training and development.

78
Q

According to the ASCA National Model, which of the following tasks related to assisting students with post-secondary plans is an appropriate activity for a school counselor?

a. Calculating class rank and grade point average

b. Coordinating schoolwide SAT testing

c. Assisting students with understanding course selections

d. Copying records needed for college applications

A

c. Assisting students with understanding course selections

Assisting students with course selections to help students make choices that are rigorous and
align with their post-secondary plans is an appropriate activity for a school counselor. Understanding appropriate and inappropriate counseling tasks is a major focus of the Management section of the ASCA National Model; this is an area that school counselors should be very familiar with in order to plan their use of time and advocate for appropriate duties. A comprehensive school counseling program designed with the ASCA National Model will focus on prevention, intervention,
and advocacy, not clerical or administrative tasks. When it comes to assisting students with post-
secondary plans, there are many tasks that school counselors may have done historically or may
still do in some schools that are not considered appropriate activities. These include tasks such as
calculating class rank or grade point average, coordinating schoolwide SAT testing, copying student
records for college applications, or doing other clerical tasks related to college applications.

79
Q

As a high school counselor, you observe that the numbers of pregnant teens on campus
have been increasing over the past few years. What steps can you put into place to address
this issue?

a. Continue to focus on student academics

b. Allow pregnant students to have a lighter course load in order to focus on their health and the pregnancy

c. Develop prevention and intervention programs to address this issue

d. Counsel students on the difficulties of having a child at this age and assist with federal aid
paperwork

A

c. Develop prevention and intervention programs to address this issue

Programs addressing teen pregnancy are designed to offer students education and support for various issues. As a counselor, regardless of your personal beliefs, you must be sensitive to the needs and views of your students. Prevention programs can provide education about safe sex, as well as information about the reality of being pregnant and a teenage parent. These prevention
programs should be geared toward those who are not pregnant but may currently be or thinking about being sexually active. Intervention programs should be geared toward those who are currently pregnant or recently had a child. These programs should provide supportive services
along with education about being a teenage parent. Whenever possible, involving the student’s parents in the programs will help increase the student’s support system.

80
Q

A school counselor is completing a curriculum results report to evaluate the effectiveness of their classroom guidance program. Which of the following does NOT correctly describe a type of data the counselor would utilize in completing this evaluation?

a. Process data describing the number of classroom guidance lessons completed and the dates
of each lesson

b. Perception data collected through feedback surveys after classroom guidance lessons

c. Outcome data describing the number of students who completed each classroom guidance
lesson

d. Outcome data describing the improvement in academic achievement as a result of
classroom guidance lessons

A

c. Outcome data describing the number of students who completed each classroom guidance
lesson

Process data focuses on the implementation of an intervention, such as the number of students who participated or the evidence of the intervention itself. Perception data describes what the participants of an intervention believe or think they have gained as a result of the intervention. Outcome data describes the measurable impact of an intervention, such as a gain in skills or
abilities or improvements toward a specific goal. Answer choice C describes process data, not
outcome data.

81
Q

Which of the following statements about English language learners (ELLs) is NOT true?

a. Typically, ELLs lag behind their English proficient peers in reading.

b. ELL students are not eligible for special education services with an IEP.

c. Spanish is the most common home language for ELLs nationwide.

d. Urban schools typically have a higher percentage of ELLs than suburban areas

A

b. ELL students are not eligible for special education services with an IEP.

While students may not be identified for an IEP based on their language needs, it is possible for a student who is identified as an English language learner to also have a disability that qualifies them for an IEP. Each of the other statements about ELLs is true and is an important consideration
for school counselors in considering the needs of their students.

82
Q

A 5-year-old boy continues to attempt to play with children who openly ridicule him and
leave him out of games. His persistence with this group of children has recently resulted in a
number of fights on the playground. After being referred to your office, you learn that he has an unsupportive home life. His parents were divorced last year. He lives with his mother who works at night, and he does not see his father. He is often left at home alone; however, a
neighbor watches him from time to time.

According to Pavlov’s theory of development, what would you expect to occur in terms of
this child’s learning and behavior?

a. Nothing, he would naturally continue to try to fit in.

b. The child would become conditioned to the negative response and avoid the situation.

c. The child would take cues from his social environment and act accordingly.

d. The child would grow out of this stage and soon make friends.

A

b. The child would become conditioned to the negative response and avoid the situation.

Pavlov’s theory of conditioning responses states that when a response repeatedly follows an action, the one performing the action will either continue or stop doing so depending on the response. In this case, the child continually receives a negative response from his peers.
Theoretically, the child would begin to relate his actions to this negative response and cease the action. However, this is not occurring and may be an indication of developmental delays. As seen in
the vignette, the child has little social support at home, and there is a clear indication of other interpersonal issues with his parents. To help this child, a counselor could explore social development delays he has and propose some type of intervention.

83
Q

In peer mediation programs, students are trained to guide peers through a conflict
resolution process based on which of the following conflict resolution styles?

a. Competing

b. Collaborating

c. Avoiding

d. Accommodating

A

b. Collaborating

Peer mediation programs train student mediators to guide their peers through a collaborative conflict resolution process. Described as a “win-win” style, this conflict resolution
style attempts to find a common solution that both parties can agree on. Other styles of conflict resolution may result in both parties not agreeing on the outcome. The competing style forces one member of the conflict to “win” at the expense of others, while the accommodating style sees one member of the conflict give in to the needs of the other member over their own. The avoiding style is exactly what it sounds like, and the conflict is ignored or postponed rather than addressed.

84
Q

Ms. McCoy is leading a counseling group for children whose parents are going through a divorce or separation. Which of the following would NOT be a goal of this type of support group?

a. Allowing children to share their feelings about their family situation

b. Allowing children to discuss the changes that are happening in their family

c. Allowing children to be reassured and feel hopeful about their situation

d. Allowing children to practice conflict resolution strategies and communication skills

A

d. Allowing children to practice conflict resolution strategies and communication skills

Skill practice is typically not a stated goal of a support group and would not be appropriate in
this situation. Skill practice would be more appropriate for an educational or therapy group.

85
Q

Mrs. Martin, a school counselor, is analyzing data about her school counseling program to identify areas for program improvement. She is reviewing perception data from a series of classroom guidance lessons delivered to 5th graders. Which of the following questions could be answered by analyzing this data?

a. Did all students receive the same curriculum?

b. What impact did the lessons have on student GPAs?

c. Did the pre- and post-tests indicate an increase in knowledge related to the lesson objectives?

d. Were the lessons delivered as planned?

A

c. Did the pre- and post-tests indicate an increase in knowledge related to the lesson objectives?

Perception data is data that measures what participants think they know, believe, or can do.
This type of data is typically measured in the form of feedback surveys or pre- and post-tests. This
is not the only type of data that can be analyzed to determine the effectiveness of a classroom
guidance curriculum and identify areas for improvement. Answer choices A and D represent
process data, and answer choice B represents outcome data. A thorough evaluation of the classroom guidance curriculum would include each of these types of data to identify both the effectiveness of the program and any potential areas of improvement.

86
Q

Which of the following is NOT true about mentor-mentee programs for students?

a. Mentor-mentee programs have a positive impact on student academic achievement.

b. Mentor-mentee programs are a tier I intervention for students at risk.

c. Mentor-mentee programs require parent permission for students to participate.

d. Mentor-mentee programs have a positive impact on the number of disciplinary referrals

A

b. Mentor-mentee programs are a tier I intervention for students at risk.

Mentor-mentee programs are a well-researched intervention for students at-risk. Tier I interventions are school-wide programs designed to provide quality instruction and support for all
students. “At-risk” students, such as those who are failing courses, have chronic absenteeism, were
retained in the previous grade level, or who have various other risk factors would most likely
require a tier II or tier III intervention, typically in a small group or one-on-one setting. Therefore, a mentor-mentee program designed for at-risk students would not be a tier I intervention. Research supports mentor-mentee programs as having positive impacts on student outcomes, both academic and behavioral. Mentor-mentee programs can be found both in the community and within schools. School counselors wishing to implement mentor-mentee programs could design and manage their
own program or seek out a community resource for their students; regardless of where the
program is managed, counselors should ensure parent permission for students to participate, especially when volunteers from the community are serving as mentors or when mentors may seek information about their mentees’ progress in school.

87
Q

Counselors have a professional responsibility to provide parents with what type of information?

a. The details of sessions with their children

b. Objective reports with respect to ethical guidelines

c. Subjective reports on their child’s progress

d. Positive parenting information

A

b. Objective reports with respect to ethical guidelines

Students have the right to seek confidential counseling. Counselors should inform parents of
this confidentiality, as well as the roles, responsibilities, and expectations of the counselor and the
student being counseled. Counselors should also work to provide parents with objective
information regarding their children within the scope of confidentiality. At times, it may be
necessary to inform the parents of certain issues pertaining to safety. This information will be
provided only after the counselor has made a professional judgment that the client is in danger or
intends to inflict harm on another and after telling the client of their intention to inform their
parents. Finally, the counselor also has a responsibility to keep conversations he has with the
parents confidential.

88
Q

When developing a school’s counseling programs, a counselor should consider what students would like to gain from the programs. Which of the following are factors important to students in regard to school counseling?

a. Academic counseling

b. Available resources and personnel

c. College-preparation assistance

d. All of the above

A

d. All of the above

When developing a counseling program, the counselor should take a number of factors into consideration, including what the students would like to gain from the programs. One factor that students often consider is academic counseling. This type of counseling provides students with
additional support as they progress through their courses, as well as support during transitional periods such as moving from high school to college. In addition to this, students are often interested in counseling programs pertaining to career and self-awareness. Many students would also like to know the availability of counselors and the frequency of planning sessions they have with the
counselor. Finally, visibility of counselors and their ability and availability to answer questions and
assist with problems is often listed as a student concern for counseling programs.

89
Q

The school data profile is a spreadsheet template that school counselors can utilize to track which of the following data points?

a. Disaggregated achievement data

b. Attainment of competencies data

c. Counseling process data

d. Needs assessment data

A

a. Disaggregated achievement data

The school data profile is an important tool provided within the ASCA National Model to
assist counselors in disaggregating student data. This data can be collected both short- and long-
term and in a variety of areas, including achievement and behavioral data. The school data profile can be used to both manage the school counseling program (by identifying areas of focus for
closing-the-gap action plans or advocating for equity issues) and evaluate the school counseling
program (by analyzing long-term data to understand the impact of the program). Attainment of competencies and needs assessment data are types of perception data that can be difficult to
represent in a spreadsheet, especially if the data is more qualitative in nature. Attainment of competencies data is process data that could be prepared in a spreadsheet but is not the focus of the school data profile.

90
Q

Which of the following would be considered an educational group?

a. A group focused on processing grief and loss.

b. A group focused on teaching parenting skills.

c. A group focused on planning a school event.

d. A group focused on overcoming test anxiety.

A

b. A group focused on teaching parenting skills.

An educational group is one that is designed to teach information and develop skills, regardless of the topic. Therefore, a group that teaches parenting skills would be considered an educational group. A group that focuses on processing a common experience of grief and loss would be an example of a support group, while the group planning a school event would be a task group. Finally, the group focused on overcoming test anxiety would be considered a counseling or therapy group.

91
Q

Cognitive behavioral therapy relies on Albert Ellis’s ABC model (originally developed as part of rational-emotive behavior therapy). What is the counselor analyzing when using the ABC model?

a. The client’s environment

b. The client’s emotional state

c. The client’s thoughts or beliefs

d. The client’s behaviors

A

c. The client’s thoughts or beliefs

Cognitive behavioral therapy is rooted in the belief that a client’s emotional state is directly
related to their beliefs or thoughts about their problems. Much of a client’s time within cognitive
behavioral therapy is spent challenging cognitions and changing self-talk, which in turn results in
changing the client’s emotional state. Don’t confuse the ABC model in cognitive behavioral therapy
with the ABC behavior model often used in schools when developing behavior interventions.

92
Q

Which of the following is true about the role of a School Counselor Performance Appraisal in an ASCA National Model program?

a. The School Counselor Performance Appraisal is not part of an ASCA National Model program, but is a licensure requirement in many states.

b. The School Counselor Performance Appraisal is a self-evaluation completed by the school
counselor once a year.

c. The School Counselor Performance Appraisal is conducted by an administrator once a year
using school, district, or state guidelines.

d. The School Counselor Performance Appraisal is based on an administrator’s observations of
the school counselor.

A

c. The School Counselor Performance Appraisal is conducted by an administrator once a year using school, district, or state guidelines.

The School Counselor Performance Appraisal is a part of the Accountability component of an
ASCA National Model program. The specifics of the tools used to complete the appraisal are based
on school, district, or state guidelines and processes. ASCA recommends that School Counselor Performance Appraisals include a self-evaluation component in addition to an administrative evaluation and assessment of goal attainment. Like all program evaluation tools, the School Counselor Performance Appraisal should be based on a variety of data. This tool can be utilized by the school counselor and school administrators to make program decisions and adjustments.

93
Q

Mrs. Simon is in her second session of individual counseling with a student who wishes to improve her grades. In their first session, the student identified her goal as completing her homework each night and turning it in each day without skipping any assignments;
previously, the student had a very poor homework completion rate. Based on brief solution-focused counseling theory, which of the following would Mrs. Simon use to begin their second session?

a. Asking the student what challenges she faced in meeting her goal over the last week.

b. Asking the student what is going better since they met last week.

c. Asking the student what she was thinking as she completed her homework each night.

d. Printing off the student’s online grade book and highlighting missing assignments.

A

b. Asking the student what is going better since they met last week.

Brief solution-focused counseling always focuses on the exception to the problem,
improvements since the previous session, and progress toward the goal. Therefore, most
subsequent sessions will begin with the counselor asking the client about what has improved since
the previous session.

94
Q

According to ASCA, which of the following is an appropriate use of standardized testing results?

a. Using standardized test scores to determine students’ eligibility to enroll in advanced coursework

b. Disaggregating standardized test scores to guide programming and instructional decisions

c. Basing promotion and retention decisions on standardized test scores

d. Determining a student’s eligibility for Section 504 services based on their standardized test
scores

A

b. Disaggregating standardized test scores to guide programming and instructional decisions

Standardized tests are a common experience and data point for schools across the country, and many schools and districts have used the results of these tests in all of the ways described in the answer choices. However, ASCA warns against placing too strong of an emphasis on individual test scores. Standardized tests provide only one data point in making decisions about curriculum,
instruction, and the education of individual students. School counselors advocate for the use of
multiple measures of student progress in making decisions about advanced or remedial coursework, intervention services, promotion, and retention. School counselors also advocate for the use of multiple measures in guiding school improvement, curriculum development, or instructional coaching. Standardized test scores can provide valuable information about academic achievement or school improvement, especially when the data is disaggregated by subgroup or compared to a norm-referenced group. However, this data should not be the sole or even primary data point in making decisions about students.

95
Q

Mr. Golden is a school counselor whose religious beliefs discourage same-sex relationships. A student shares in individual counseling that he is in a same-sex relationship.
What is Mr. Golden’s ethical obligation in working with this student?

a. Mr. Golden should refer the student to a different counselor within the school due to the
bias his religious beliefs create.

b. Mr. Golden should share his bias with the student so that the student has a full understanding before giving informed consent.

c. Mr. Golden should seek training and development in this area to help him provide comprehensive services to the student.

d. Mr. Golden should terminate the counseling relationship with the student as his beliefs are
constitutionally protected.

A

c. Mr. Golden should seek training and development in this area to help him provide comprehensive services to the student.

According to the ASCA Ethical Standards for School Counselors, counselors should evaluate
any personal biases that may prevent them from providing comprehensive services to all students.
When school counselors identify biases that interfere with providing comprehensive services to all
students, they should seek additional training and development rather than deny counseling
services to these students based on their personal biases. School counselors should work to
promote an environment that respects diversity.

96
Q

Which of the following is NOT a type of results report completed as part of a school counseling program evaluation?

a. Curriculum results report

b. Individual results report

c. Small-group results report

d. Closing-the-gap results report

A

b. Individual results report

There are three types of action plans that can be created as part of a comprehensive school
counseling program: curriculum action plan, small-group action plan, and closing-the-gap action
plan. Therefore, the three corresponding results reports to analyze the effectiveness of these action
plans are the curriculum results report, small-group results report, and closing-the-gap results
report. There is not an individual counseling action plan or results report.

97
Q

There are generally six levels of parent participation. Which of the following is not considered a level of parent participation?

a. Parenting

b. Communication

c. Continued Education

d. Volunteering

A

c. Continued Education

While students with educated parents are not a risk factor, this is not considered a type of parent involvement. Involvement begins at the parenting level. Schools cannot tell individuals how to parent; however, they can provide information for parents about encouraging good academic
practices while at home. For example, informing parents about the appropriate amount of time their child should spend on homework is one strategy. Communication is the next level of participation. Sending home progress reports and holding conferences both increase communication between schools and parents. The third level involves volunteering, which can include one-time projects or long-term participation. The last three levels include learning at home. Examples of this are providing information on the skills their children will learn throughout the school year, decision-making by encouraging participating on a school advisory board, and collaboration with the community.

98
Q

A counselor initiating a school-family curriculum program will probably structure the program in what way?

a. Take-home activities for students and their parents

b. Group discussions

c. Surveys for parents

d. Counseling sessions for those in need of cultural diversity education

A

b. Group discussions

Discussion groups are effective ways to get parents and students involved in learning about cultural diversity. These discussion groups can offer participants the opportunity to talk about their particular cultures. Additionally, these groups offer a nonthreatening venue for others to ask questions and work out misunderstandings about various cultures that have been perpetuated by stereotypes. These groups also offer parents and students the chance to celebrate different cultures
though activities. In addition to group discussions about participants’ cultures, discussions about
various types of families and lifestyles are beneficial. For example, biracial families, families with adopted or foster children, and families with stepparents are increasingly common in American society, and education about these various types of families is just as important as learning about
other cultures.

99
Q

Young girls who experience abuse in the home or in a relationship have increased risk factors including:

a. Drug abuse

b. Teen pregnancy

c. Poor grades

d. All of the above

A

d. All of the above

Regardless of the type of abuse a girl may experience, there will be deep psychological effects
that may last much longer than the actual abuse. Oftentimes, these girls will have low self-esteem
and look for ways to make themselves feel better. Additionally, those children who are abused may
turn to drugs and alcohol to numb the effects of the abuse. Teenage girls who experience abuse are
also more likely to become pregnant than those girls who do not experience abuse. Finally, those
experiencing abuse will often lose interest in school and their futures, and counselors will see a
decline in their grades. Since the idea of abuse if often thought of taboo and embarrassing,
counselors should be aware of these risk factors and outcomes when assessing students.

100
Q

A counselor begins seeing a student who has issues with low self-esteem. This student is of Chinese descent but was adopted by an African American family as a baby. What
responsibility does the counselor have toward the student regarding culture?

a. Provide the student with information on her Chinese heritage and encourage her to embrace it

b. Suggest she join the Chinese American Club at school

c. Question her feelings of being of Chinese descent but being raised in an African American home

d. Respect the student’s cultural identity as it is and assist her instead with the problems she brought to the attention of the counselor

A

d. Respect the student’s cultural identity as it is and assist her instead with the problems she brought to the attention of the counselor

Counselors must respect the cultural identity of their students. Since the student is not seeing the counselor because of cultural identity issues, it would be considered professionally
unethical for the counselor to address her Chinese heritage. It is, however, the counselor’s responsibility to understand this student’s cultural identity in order to develop effective interventions for this student’s presenting problem. The counselor must also have a clear
understanding of her own beliefs and stereotypes in this situation that may cause bias when treating this student. Counselors who are unaware of these issues may inadvertently treat a student
of another culture with negativity or less support than she would a student of her own culture.

101
Q

A school counselor is creating a spreadsheet of student achievement data that includes end-of-grade test scores, graduation rates, and enrollment in honors courses. The counselor disaggregates the data by student subgroup. The counselor is most likely compiling this data
for which of the following tasks?

a. Creating a closing-the-gap action plan

b. Creating a small-group action plan

c. Preparing the annual agreement

d. Conducting a needs assessment

A

a. Creating a closing-the-gap action plan

Thorough data analysis, including disaggregating data by subgroup, is typically used for either creating a closing-the-gap action plan or completing a school data profile. This type of data analysis is helpful for identifying potential achievement gaps or equity issues within the school or district that the counselor can plan to address with a closing-the-gap action plan. This type of
detailed, schoolwide data analysis is typically not used for creating a small-group action plan, although a small group may be utilized as a strategy within a closing-the-gap action plan. The annual agreement is a document created between a counselor and an administrator to plan for and establish goals for the upcoming school year and is focused on the counseling program itself; a
needs assessment is usually in the form of a survey completed by students, staff, and/or parents to
identify strengths and areas of concern in the school.

102
Q

A school counselor implements a classroom guidance program aimed at improving graduation rates. During the annual program evaluation, the counselor analyzes a variety of data to determine the effectiveness of the program and implement changes for the following
year. Which of the following pieces of data gathered by the counselor represents perception data?

a. 450 students in grades 9-12 completed three classroom lessons with their counselor.

b. 120 9th-grade students submitted complete four-year graduation plans to their counselor.

c. 135 students indicated they feel “very prepared” to complete high school in four years.

d. The school’s current four-year graduation rate is 83%, an increase of 7% over last year.

A

c. 135 students indicated they feel “very prepared” to complete high school in four years.

With programs aimed at increasing graduation rates, outcome data such as the four-year graduation rate is often the primary piece of data that is highlighted; an effective counselor
examines graduation rates even further by disaggregating data into student subgroups in order to
identify gaps in achievement. Changes in this rate over time are also important in identifying areas
of need or improvements made as a result of school counseling programs. School counselors also

103
Q

Collaboration with other teachers is a good way to incorporate positive behaviors and communication throughout the school. How can a teacher advisor program assist in this collaboration?

a. Provides all-around support to the students

b. Makes the teacher feel important and more willing to work with the counselor

c. Provides more watchful eyes for bad behavior

d. Gives students more opportunities to get help with their school work

A

a. Provides all-around support to the students

Teacher advisors cooperate with school counselors to provide students with all-around
support. These advisors help students adjust to a new school by offering orientation and support.
Additionally, teacher advisors provide students with a sounding board when resolving disputes or
misunderstandings with faculty members or other students. Oftentimes, advisors will initiate
student work groups for various issues students may experience. These groups may focus on
communication, conflict resolution, or general adjustment strategies. Overall, teacher advisors
serve as advocates for students in order to make school and the educational experience both
pleasant and rewarding.

104
Q

Rather than requiring students who disobey the rules on fighting to serve a silent detention, you suggest conducting a problem-centered, structured intervention group. What
issues should you focus on during this group?

a. Self-actualization

b. Aggressive behaviors

c. Safety

d. Physiological issues

A

b. Aggressive behaviors

Problem-centered, structured counseling groups allow students to share their experiences and hear similar experiences from their peers in a safe environment. This type of group setting also allows students to gain the support of their peers as well as receive feedback. Counselors
conducting these groups should focus on changing deviant behaviors and encouraging participants
to try out new behaviors during the week. These exercises can be discussed during sessions, and
students can learn how to brainstorm appropriate behaviors together by sharing their experiences. While it is possible to conduct a problem-centered group for a number of different issues, topics such as self-actualization, safety, and physiological issues are typically the focus of primary prevention groups.

105
Q

What is the purpose of demographic information on survey assessments?

a. No real purpose

b. This information should not be included.

c. Identify differences

d. Identify similarities

A

c. Identify differences

Typically, a counselor will conduct a student survey in order to gain an overall view of the student body. By collecting demographic information, the counselor can determine important differences in the student body. This information will assist in developing additional programs,
activities, or interventions. When conducted as a formative assessment, demographic information can provide an indication of whether different groups of students are finding more benefit in a
particular program or intervention. Again, this information will allow the counselor to modify a program or add programs to better fit the needs of all students involved.

106
Q

Which of the following questions is answered by the school counseling program evaluation?

a. “How are students different as a result of the school counseling program?”

b. “What measurable progress have students made toward counseling program objectives?”

c. “How effective is the school counseling program in achieving its stated goals?”

d. “What impact has the school counseling program had on academic achievement?”

A

a. “How are students different as a result of the school counseling program?”

The ASCA National Model states the question to be answered by the program evaluation as “How are students different as a result of the school counseling program?” Within the program evaluation, data about academic achievement, progress toward stated goals and objectives,
counselor use of time, and the effectiveness of program components is collected and evaluated, but the impact on students is the focus of the evaluation process. Measuring student impact allows the
school counselor to understand their progress toward stated goals and to identify areas for improvement or adjustments to their counseling program.

107
Q

In designing and implementing a comprehensive school counseling program, which of the following is NOT a type of action plan utilized by the ASCA National Model?

a. Curriculum action plan

b. Small-group action plan

c. Closing-the-gap action plan

d. Use-of-time action plan

A

d. Use-of-time action plan

The use-of-time assessment is an ASCA National Model document, but it is not an action plan. The use-of-time assessment analyzes the counselor’s activities over the course of the school year and is typically done twice per year; as an assessment, it is a look backward over activities already conducted rather than a data-driven plan for future actions. Curriculum action plans, small-group action plans, and closing-the-gap action plans are all data-driven planning documents that
help the counselor design interventions for the future. The curriculum action plan helps counselors create an effective plan for implementing the school counseling curriculum, and the small-group action plan is a similar document for planning small-group interventions. Closing-the-gap action plans help counselors design interventions focused on addressing discrepancies in academic or
behavioral outcome data between student groups.

108
Q

What is the best strategy for choosing teacher advisors for students?

a. Divide the students alphabetically by last name

b. Obtain a list of students each teacher would like to advise

c. Obtain a list of preferred teacher advisors from the students

d. Obtain a list of preferred teacher advisors from the parents

A

c. Obtain a list of preferred teacher advisors from the students

Teacher advisors serve as advocates for students. They are respected adults whom students can turn to for support when problems arise. Therefore, it is important for students to be comfortable with their teacher advisors. While it is not practical to allow students to make a final
decision on the advisor, schools should include students in the decision-making process. It is
suggested that students be able to choose three to five teachers they would like to work with. Based
on this information, counselors and teachers can then assign teacher advisors while keeping the
student’s wishes in mind.

109
Q

Mrs. Kim is planning to conduct a social skills group for students who demonstrate difficulty with peer interactions and aggressive behavior. She begins her group selection process by asking teachers to identify which students they feel would benefit from the group. What would be the next step in the group selection process?

a. Conduct a brief individual interview with each of the students to determine if they have the
social skills to participate in the group effectively

b. Ask the identified students what they would like to achieve as part of their group participation

c. Send home permission slips to the parents of the identified students to gain parental consent for group participation

d. Conduct an individual counseling session with each of the identified students to practice the
skills needed to participate in the group

A

a. Conduct a brief individual interview with each of the students to determine if they have the
social skills to participate in the group effectively

Particularly with a group focused on social skills with students who struggle with appropriate social behavior, the individual interview is an extremely important part of the group selection process. Since all of the identified students are already known to struggle with social
skills, it is necessary to interview each student to determine their current level of functioning with the skills necessary for group participation, such as active listening and turn-taking. Answer D, conducting an individual counseling session to practice group skills, may be appropriate for some group members after determining the final makeup of the group.

110
Q

A school counselor working to create an academic intervention plan for a student is most likely to utilize which of the following approaches to develop the plan?

a. Strengths-based focus

b. Problem identification

c. Cognitive behavioral therapy

d. Family therapy

A

a. Strengths-based focus

The recommended and most commonly used practice for academic intervention plans is a
strengths-based focus. Most students needing an academic intervention plan are already aware (or their parents and teachers are aware) of the problems they are facing. The school counselor’s role in the academic intervention process is to identify the strengths the student has that may be supported in order to increase academic outcomes. The interventions designed in this type of plan are typically focused on either utilizing existing strengths or overcoming barriers. Cognitive
behavioral therapy describes a method of one-on-one counseling, which at times may be a part of
the plan, but it would not be the approach used to actually develop the plan. Family factors are often considered in developing academic intervention plans, but family therapy is not part of a school counselor’s duties and should not be used to develop an academic intervention plan.

111
Q

Which of the following serves as an initial coping mechanism when experiencing grief and loss?

a. Anger

b. Depression

c. Denial

d. Bargaining

A

c. Denial

Denial occurs when individuals experience shock and disbelief over the loss of a loved one. The experience is so overwhelming that it cannot be experienced all at once. Denial serves as a protective or coping mechanism during the initial stage of grief and loss. Elizabeth Kubler-Ross’s five stages of grief are denial, anger, bargaining, depression, and acceptance. Individuals in the
anger stage of grief may blame others or believe that the circumstances surrounding the death were unfair, whereas individuals in the bargaining stage may try to make a deal with a higher power to
have a better or different outcome. The depression stage of grief involves profound sadness and loss before finally reaching the acceptance stage in which the individual accepts the permanently altered reality and readjusts to live life beyond the loss.

112
Q

In survey-type assessments, a Likert-type scale is often used. These scales use either an even or an odd number of responses. For scales using an even number of responses, there is no neutral answer, and the individual must choose either a negative or a positive response. For
scales using an odd number of responses, the individual has the opportunity to choose a neutral or undecided response.

A student is given a 5-point Likert-type scale and selects all Neutral responses. What would a counselor determine from these results?

a. The student is laid back and easy going.

b. The student did not want to share his own thoughts and wanted to get the assessment done as quickly as possible.

c. The student did not understand the task.

d. The student is in need of intervention for indecisiveness.

A

b. The student did not want to share his own thoughts and wanted to get the assessment done as quickly as possible.

Although using this type of assessment may result in more neutral responses, most individuals will provide their true thoughts. Receiving an assessment with all neutral responses should be considered as an outlier. Depending on the purpose of the assessment, the counselor may consider disregarding it in the results and providing an explanation in any required report. Depending on the nature of the assessment, the counselor can conduct additional assessments with the student to determine any difficulties this student may be having. Regardless, when giving students surveys, it must be indicated that all responses are confidential, and the students should
respond as honestly as possible.

113
Q

Hanna is a new kindergarten student who is legally blind. Her parents talk to the school counselor during the enrollment process and ask for information about the services and
supports their daughter will be able to access in school. Which of the following laws or programs would the counselor refer to in order to provide guidance about Hanna’s
situation?

a. IDEA

b. Section 504

c. FERPA

d. ESSA

A

a. IDEA

IDEA, the Individuals with Disabilities Education Act, provides for the education of children
with disabilities by requiring schools to provide a free, appropriate public instruction through an individualized education plan (IEP) to children with disabilities in one of thirteen categories. These
categories include: autism, deaf-blind, developmental delay, emotional disturbance, hearing
impairments (including deafness), intellectual disability, multiple disabilities, orthopedic
impairments, other health impairments, specific learning disabilities, speech or language
impairments, traumatic brain injury, and visual impairments (including blindness). Therefore, the
school counselor would refer to the provisions of this law as well as school policies under IDEA to
provide information to Hanna’s parents. The other laws named in answer choices include Section
504, Family Educational Rights and Privacy Act (FERPA), and Every Student Succeeds Act (ESSA).
Section 504 is also a disability-related law, but Section 504 addresses how a school will provide
accommodations and modifications to remove the barriers a child’s disability may create for
accessing curriculum; an IEP outlines a specific educational plan including specialized instruction.
FERPA has to do with the privacy of student records, and ESSA is a federal law guiding K-12 public
education policy.

114
Q

A school counselor is planning a classroom guidance lesson on career development. Based on psychological career theory, what would be an appropriate first activity to include in this lesson?

a. Having students research careers they are already interested in

b. Having students complete a survey to determine their personality type

c. Taking a field trip to visit a real-life work setting

d. Having students research the skills and abilities needed for various careers

A

b. Having students complete a survey to determine their personality type

Psychological career theory, also known as personality career theory, focuses on an individual’s personality type and the types of careers that best fit that personality type. A
psychological or personality approach will always begin with a Self-Directed Search to determine
the individual’s personality type. Researching careers that students are already interested in or
learning about the skills and abilities needed in those careers are common activities in many schools, but these do not fit the psychological career theory whereby individuals are matched to potential careers based on their personality type. Before researching careers, students would need to take the personality survey to determine which types of careers are best suited for their personality type. A field trip to visit a real-life work setting would be more aligned with a social learning career theory, such as the approach developed by John Krumboltz. A social learning career
theory is based on the concept that interests are learned, so exposure to a variety of careers and work sites gives an individual more learning and experiences from which to develop their interests.

115
Q

A middle school counselor receives a phone call from a case manager at a community agency. The agency provides tutoring and enrichment opportunities for low-income students, and the case manager tells the counselor that they are working with several students at the counselor’s school and would like to know how they are progressing
academically as a result of their tutoring. Which of the following would be the most appropriate way for the school counselor to work with this agency?

a. Tell the case manager that they will have to ask the students’ parents for this information, as FERPA prohibits the counselor from sharing academic information with an outside agency.

b. Provide the students’ parents with a release of information form which, once signed, will permit the counselor and case manager to share certain information with each other.

c. Share basic information with the case manager as asked because they represent a stakeholder with a “legitimate educational interest” in the students.

d. Send printed copies of the information with the students to provide to the agency so that
they can decide what they would like to share.

A

b. Provide the students’ parents with a release of information form which, once signed, will permit the counselor and case manager to share certain information with each other.

Although FERPA permits sharing academic information within the school for staff with a “legitimate educational interest,” outside agencies do not qualify for this permission. Although sharing this information with the case manager would probably directly benefit the student by assisting the tutoring program with identifying areas of need, the counselor must have parental consent to share this information with any outside agency. Having the student choose whether to provide this information is not appropriate, especially if they are a minor.

116
Q

A supervisor for a counseling program collects outcomes data for all clients who received services last year. When plotting the distribution, the supervisor notices that clients younger than age 12 have measures skewed to the left. Which of the following is true of distributions skewed to the left (i.e., negatively skewed)?

a. The mean is less than the median.

b. The mean is greater than the median.

c. The mean and the median are the same value.

d. The mode is the lowest value.

A

b. The mean is greater than the median.

Generally speaking, the mean is less than the median in distributions with data skewed to
the left. The mean, median, and mode (i.e., measures of central tendency) are all the same values in
a normal distribution. When distributions are skewed to the right, the mode is the lowest value. In
distributions skewed right (i.e., positive distributions), the mode is less than the median and the
median is less than the mean.

117
Q

Which of the following statements is true regarding electronic communications according to the ASCA Ethical Standards for School Counselors?

a. Electronic communication between the school counselor and other individuals represents a
form of privileged communication and is confidential.

b. Electronic communication between school officials about an individual student can create
an official school record under FERPA.

c. Electronic communication about an individual student does not create an official school
record under FERPA as long as the student’s name is not used.

d. The only form of electronic communication that constitutes an official school record is that
which takes place within the official student information system (SIS).

A

b. Electronic communication between school officials about an individual student can create
an official school record under FERPA.

Any form of electronic communication between the school counselor and other school
officials can create an official school record that needs to be treated appropriately under FERPA, even if student names are not used. All forms of electronic communication should adhere to the ethical standards, and special care should be taken to protect confidentiality by utilizing accepted security standards. The ASCA Ethical Standards encourage counselors to take care in purging electronic sole-possession case notes, similar to hard-copy sole possession case notes.

118
Q

Classroom guidance is most often considered to be which type of counseling activity?

a. Prevention

b. Intervention

c. Outreach

d. Academic

A

a. Prevention

Generally speaking, classroom guidance is a preventative activity; it is systematically delivered to all students. Intervention activities take place when an issue or problem is identified, therefore an activity that is systematically delivered to all students is not an intervention activity. Outreach involves literally “reaching out” of the school and into the community, and classroom guidance obviously does not fit this description. While classroom guidance may be focused on academic topics, “academic” is not a type of counseling activity.

119
Q

What are three essential developmental skills?

a. Self-concept, self-esteem, self-respect

b. Ego, Id, Super-ego

c. Operational thinking, concrete thinking, abstract thinking

d. Thinking, feeling, relating

A

d. Thinking, feeling, relating

Thinking, feeling, and relating are considered a modern model of human development. Thinking begins at home and consists of the values, thoughts, and beliefs imposed by parents. These thoughts are further supported by social interactions. Cognitive skills training assists students in further developing these abilities. Feelings can also be associated with the development of self-concept and self-esteem. Emotional literacy skills training and programs help students become
more aware of their own feelings. Finally, relating involves an individual’s interpersonal skills.
Social literacy skills programs will assist students in relating to other students and adults.

120
Q

Which of the following types of data would be best utilized to identify areas for program improvement in an annual school counseling program evaluation?

a. Closing-the-gap action plan

b. School data profile

c. State testing results

d. Program outcome data

A

d. Program outcome data

Program outcome data provides information on the direct impacts of the school counseling program on student outcomes. When combined with program process and perception data, outcome data helps a school counselor create a clear picture of their program over the course of a
school year. The school data profile, closing-the-gap action plan, and state testing results can all help clarify goals for the next school year, but the outcome data provides information that is most directly related to the school counseling program to identify areas for improvement.